GT 8

You might also like

Download as pdf or txt
Download as pdf or txt
You are on page 1of 176

6/9/24, 12:45 PM Neet PG Preparation, Neet PG Coaching, FMGE, USMLE

Grand Test-8 (Clinical New Pattern)

RESULT

View View View SectionWise


Result
leaderboard Analysis Solution Analysis

Grand Test-8 (Clinical New Pattern)


Question Wise Report

Question: 1
Saturated solution of NaCl is used as a preservative for viscera in

A Corrosive poisoning

B Aconite

C Corrosive sublimate

D Phosphorous

13.09% People got this right

Explanation:

Correct Answer (D)

Best preservative of viscera is Rectified Spirit.

Saturated solution of sodium chloride is not used in Corrosives, Aconite and


Corrosive sublimate poisoning.

https://emedicoz.com/testresult/683510s14693192 1/176
6/9/24, 12:45 PM Neet PG Preparation, Neet PG Coaching, FMGE, USMLE

Question: 2
What is the punishment that can be given to the doctor for sex
determination under PCPNDT act?

A 3 yrs and Rs 10000/-

B 5 yrs and Rs 10000/-

C 3 yrs and Rs 100000/-

D 5 yrs and Rs 100000/-

20.46% People got this right

Explanation:

Correct Answer (A)

PCPNDT ACT - Punishment

1st offence - 3 yrs and Rs 10000/-

Subsequent offence – 5 yrs and Rs 50000/-

Question: 3
Which of the following is not a method of crime scene investigation?

A Grid

B Strip

C Point to point

https://emedicoz.com/testresult/683510s14693192 2/176
6/9/24, 12:45 PM Neet PG Preparation, Neet PG Coaching, FMGE, USMLE

D Composite

16.87% People got this right

Explanation:

Correct Answer (D)

Question: 4
Most common hallucinations in cocaine poisoning are

A Auditory

B Tactile

C Visual

D Gustatory

10.45% People got this right

Explanation:

Correct Answer (C)

Visual hallucinations are more common than tactile hallucinations.

Tactile hallucinations are known as Magnan’s symptoms or Cocaine bugs

Question: 5
Identify the injury

https://emedicoz.com/testresult/683510s14693192 3/176
6/9/24, 12:45 PM Neet PG Preparation, Neet PG Coaching, FMGE, USMLE

A Incised wound

B Flaying

C Chop wound

D Stab wound

56.41% People got this right

Explanation:

Correct Answer (B)

Flaying aka Avulsion laceration.

Question: 6
Which cardiac defect has the worst prognosis, without immediate surgical
correction?

A d-TGA with no associated


defects

https://emedicoz.com/testresult/683510s14693192 4/176
6/9/24, 12:45 PM Neet PG Preparation, Neet PG Coaching, FMGE, USMLE

B 1-TGA with no associated


defects

C d-TGA with large ASD

D d-TGA with restrictive VSD

30.35% People got this right

Explanation:

Correct Answer (A)

d-Transposition of the great arteries is a medical emergency. Cyanosis and


tachypnea are most often recognized within the 1st hours or days of life. If it is
not treated, the vast majority of these infants would not survive the neonatal
period. Hypoxemia is usually moderate to severe, depending on the degree of
atrial level shunting and whether the ductus is partially open or totally closed.
Infants who remain severely hypoxic or acidotic despite prostaglandin infusion
should undergo Rashkind balloon atrial septostomy. A successful Rashkind atrial
septostomy should result in a rise in PaO2 to 35-50 mm Hg and elimination of
any pressure gradient across the atrial septum. Some patients with TGA and VSD
may require balloon atrial septostomy because of poor mixing, even though the
VSD is large. If the VSD associated with d-TGA is small, the clinical lit
manifestations, laboratory findings, and treatment are similar to those described
previously for transposition with an intact ventricular septum.

When the VSD is large and not restrictive to ventricular ejection, significant
mixing of oxygenated and deoxygenated blood usually occurs and clinical
manifestations of cardiac failure are seen. The degree of cyanosis may be subtle
and sometimes may not be recognized until an oxygen saturation measurement
is performed. The physiology of I- TGA is quite different from that of d-TGA. The
double inversion of the atrioventricular and ventriculoarterial relationships result
in desaturated right atrial blood appropriately flowing to the lungs and
oxygenated pulmonary venous blood appropriately flowing to the aorta. The
circulation is thus physiologically “corrected.” Without other defects, the
hemodynamics would be nearly normal.

https://emedicoz.com/testresult/683510s14693192 5/176
6/9/24, 12:45 PM Neet PG Preparation, Neet PG Coaching, FMGE, USMLE

Question: 7
A previously normal newborn infant in a community hospital nursery is
noted to be cyanotic at 14 hours of life. She is placed on a face mask with
oxygen flowing at 10 L/min. She remains cyanotic, and her pulse oximetry
reading does not change. An arterial blood gas shows her PaO2 to be 23
mm Hg. Bilateral breath sounds are present, and she has no murmur. She
is breathing deeply and quickly, but she is not retracting. While you are
waiting for the transport team from the nearby children's hospital, you
should initiate which of the following?

A Indomethacin infusion

B Saline infusion.

C Adenosine infusion

D Prostaglandin El infusion

42.61% People got this right

Explanation:

Correct Answer (D)

The vignette describes an infant with a ductal-dependent cyanotic congenital


heart lesion. In this example, the child had pulmonary atresia without a
corresponding VSD; another example would have been transposition of the
great vessels without a septal defect to allow mixing of oxygenated and non-
oxygenated blood. Cyanotic infants who do not improve then- saturations with
supplemental oxygen should be evaluated carefully for structural heart disease.
The ductus arteriosis typically closes in the first few hours of life; thus, these
children will develop their cyanosis in the same time frame. Prostaglandin E1 will
help keep the ductus patent until a surgical procedure can be performed. PGE1
does have the tendency to cause hypoventilation, so arrangements must be

https://emedicoz.com/testresult/683510s14693192 6/176
6/9/24, 12:45 PM Neet PG Preparation, Neet PG Coaching, FMGE, USMLE

made for a potential artificial airway if necessary. Indomethacin closes a patent


ductus arteriosis. Adenosine is used for supraventricular tachycardia.

Question: 8
IVIG-resistant KD occurs in approximately …….of patients and is defined
by persistent or recrudescent fever 36 hr after completion of the initial
IVIG infusion.

A 15

B 25

C 35

D 45

27.45% People got this right

Explanation:

Correct Answer (A)

15

IVIG-resistant KD occurs in approximately 15% of patients and is defined by


persistent or recrudescent fever 36 hr after completion of the initial IVIG
infusion. Patients with IVIG resistance are at increased risk for CAA> Typically,
another dose of IVIG at 2 g/kg is administered to patients with IVIG resistance.

Question: 9
Clinical features of 11-beta hydroxylase deficiency are:

https://emedicoz.com/testresult/683510s14693192 7/176
6/9/24, 12:45 PM Neet PG Preparation, Neet PG Coaching, FMGE, USMLE

A Hypotension,
hyperkalemia, hyponatremia,
virilisation

B Hypotension, hypokalemia,
hypermatremia, virilisation

C Hypertension, virilisation

D Hypertension, feminization

34.68% People got this right

Explanation:

Correct Answer (C)

Hypertension, virilisation

Although cortisol is not synthesized efficiently, aldosterone synthetic capacity is


normal, and some corticosterone is synthesized from progesterone by the intact
aldosterone synthase enzyme. Thus, it is unusual for patients to manifest signs of
adrenal insufficiency such as hypotension, hypoglycemia, hyponatremia, and
hyperkalemia. Approximately 65% of patients become hypertensive, although
this can take several years to develop. Hypertension is probably a consequence
of elevated levels of deoxycorticosterone,, which has mineralocorticoid activity.
Infants may transiently develop signs of mineralocorticoid deficiency after
treatment with hydrocortisone is instituted. This is presumably from sudden
suppression of deoxycorticosterone secretion in a patient with atrophy of the
zona glomerulosa caused by chronic suppression of rennin activity. All signs and
symptoms of androgen excess that are found in 21- hydroxylase deficiency may
also occur in 11β-hydroxylase deficiency.

Question: 10

https://emedicoz.com/testresult/683510s14693192 8/176
6/9/24, 12:45 PM Neet PG Preparation, Neet PG Coaching, FMGE, USMLE

A 9 day old neonate, born by vaginal delivery to a 21 yr old healthy


woman, is presented with fever, lethargy, and poor feeding. There are no
diagnostic findings on physical examination, and results of sepsis
evaluation, including a CBC and cerebrospinal fluid studies, are
unremarkable. Ampicillin and cefotaxime are begun. Two days later all
bacterial cultures are negative but the child's clinical condition worsens,
with falling blood pressure, decreased level of consciousness,
thrombocytopenia, and elevated liver enzymes. The indicated change in
treatment is:

A Addition of amphotericin

B Addition of yancomycin

C Addition of ribavirin

D Addition of acyclovir

27.45% People got this right

Explanation:

Correct Answer (D)

Add acyclovir

Infants with disseminated HSV infections generally become ill at 5-11 days of
life. Their clinical picture is similar to that of infants with bacterial sepsis,
consisting of hyperthermia or hypothermia., irritability, poor feeding, and
vomiting. They may also exhibit respiratory distress, cyanosis, apneic spells,
jaundice, purpuric rash, and evidence of central nervous system infection;
seizures arc common. Skin vesicles are seen in about 75% of cases. If untreated,
the infection causes shock and disseminated intravascular coagulation;
approximately 90% of these infants die, and most survivors have severe
neurologic sequelae.

https://emedicoz.com/testresult/683510s14693192 9/176
6/9/24, 12:45 PM Neet PG Preparation, Neet PG Coaching, FMGE, USMLE

Question: 11
A 3 yrs old girl presents to your office with acute onset of lethargy and
pallor. The child's mother reports that the child had bloody diarrhea for 5
days that cleared 1 day before presenting to your office. She also notes
acute onset of cola-colored urine. On examination, the patient is pale and
lethargic. Blood pressure is 120/80 mm Hg. The most appropriate next
step in diagnosis would be:

A Urinalysis

B X-ray examination of the


abdomen

C Urine culture

D Complete blood cell count

20.29% People got this right

Explanation:

Correct Answer (D)

Do a complete blood count as this child has HUS

Question: 12
An otherwise, healthy, asymptomatic 16-yr-old African-American girl is
found to have hypertension and 3+ proteinuria by dipstick testing on
mid-day and 1st morning voided urine samples. The microscopic analysis
shows 0-2 red blood cells per high-power field. The most likely diagnosis
is:

A Postinfectious
glomerulonephritis

https://emedicoz.com/testresult/683510s14693192 10/176
6/9/24, 12:45 PM Neet PG Preparation, Neet PG Coaching, FMGE, USMLE

B Diabetic nephropathy

C Minimal change nephrotic


syndrome

D Focal segmental
glomerulosclerosis

17.39% People got this right

Explanation:

Correct Answer (D)

FSGS

Question: 13
A 6 month old infant began to vomit and ceased to gain weight. At 9
months he was again readmitted in the hospital. Routine examination and
lab tests were normal, but after one week he came drowsy, his temperature
rose to 39 degree C, and had hepatomegaly. Since the child could not take
milk, even by tube feeds, intravenous glucose was given, and he improved
his sensorium.

Urine analysis showed very high glutamine and uracil.

What is the enzyme deficiency?

A Arginase

B Carbomyl phosphate
synthetase-1

https://emedicoz.com/testresult/683510s14693192 11/176
6/9/24, 12:45 PM Neet PG Preparation, Neet PG Coaching, FMGE, USMLE

C Glutaminase

D Ornithine transcarbomylase

26.18% People got this right

Explanation:

Correct Answer (D)

Ornithine transcarbomylase

Question: 14
A 9 days old full-term infant is admitted to the hospital with lethargy,
fever, and increasing jaundice. Physical examination also reveals
hepatomegaly. Laboratory results reveal a blood glucose value of 10
mg/dL. total and direct bilirubin values of 15 and 7 mg/dL., respectively,
and liver enzymes AST, 700 units/L, and ALT, 650 units/L. The next day the
blood culture is positive for a gram-negative rod. The most likely
diagnosis is:

A Necrotizing enterocolitis

B Galactosemia

C Neonatal hepatitis

D Glycogen storage disease

15.46% People got this right

Explanation:

Correct Answer (B)

Galactosemia

https://emedicoz.com/testresult/683510s14693192 12/176
6/9/24, 12:45 PM Neet PG Preparation, Neet PG Coaching, FMGE, USMLE

The diagnosis of uridyl transferase deficiency (galactosemia) should be


considered in newborn or young infants with any of the following features:
jaundice, hepatomegaly, vomiting, hypoglycemia, seizures, lethargy, irritability,
feeding difficulties, poor weight gain or failure to regain birth weight,
aminoaciduria„ nuclear cataracts, vitreous hemorrhage, hepatic failure, liver
cirrhosis, ascites, splenomegaly, or mental retardation. Patients with
galactosemia, are at increased risk for Escherichia coli neonatal sepsis; the onset
of sepsis often precedes the diagnosis of galactosemia.

Question: 15
The best test for universal screening of metabolic disorders is:

A Blood TMS

B Blood HPLC

C Urine GCMS

D Guthrie’s test

40.73% People got this right

Explanation:

Correct Answer (A)

Blood TMS

Question: 16

https://emedicoz.com/testresult/683510s14693192 13/176
6/9/24, 12:45 PM Neet PG Preparation, Neet PG Coaching, FMGE, USMLE

What is not true regarding the condition in the picture above:

A Defective adduction of the


left eye

B Ptosis

C Light reflex absent

D Accomodation absent

26.54% People got this right

Explanation:

Correct Answer (E)

This is a third nerve palsy of the left eye. It is characterised by crossed diplopia
as the eye is down and out .

Question: 17
With the near point of 20cms, a hypermetrope of +5 has to exercise
accommodation of :

A +11D

https://emedicoz.com/testresult/683510s14693192 14/176
6/9/24, 12:45 PM Neet PG Preparation, Neet PG Coaching, FMGE, USMLE

B +9D

C +10D

D +0D

19.49% People got this right

Explanation:

Correct Answer (C)

The amplitude of accommodation is the refractive power at near point-refractive


power at far point.

Since power is 1/focal length in metres

Hence the power at near point =100/20=5D

Far point of a hypermetrope is behind the eye and depicted as negative. Hence
power of far point = -5

Amplitude of accommodation= 5-[-5]=+10D

Question: 18
True association include all except:

A Wilsons disease –
sunflower cataract

B Myotonic dystrophy—
Christmas -tree cataract

C Amiodarone—Anterior
capsular cataract

https://emedicoz.com/testresult/683510s14693192 15/176
6/9/24, 12:45 PM Neet PG Preparation, Neet PG Coaching, FMGE, USMLE

D Alports syndrome –
posterior lenticonus

41.1% People got this right

Explanation:

Correct Answer (D)

Alports syndrome is associated with anterior lenticonus .Posterior lenticonus


occurs in Lowe’s syndrome

Question: 19
A patient with a running nose and pain over the medial aspect of the eye
being treated with decongestants for many days. He later developed
chemosis, proptosis and diplopia on abduction of right eye with
congestion of the optic disc. What is the most probable diagnosis?

A Acute Ethmoidal Sinusitis

B Orbital cellulitis

C Cavernous sinus
thrombosis

D Orbital apex syndrome

21.05% People got this right

Explanation:

Correct Answer (C)

Proptosis and diplopia on abduction [sixth nerve involvement is the first sign]
with papilloedema , the most probable diagnosis is cavernous sinus thrombosis.

https://emedicoz.com/testresult/683510s14693192 16/176
6/9/24, 12:45 PM Neet PG Preparation, Neet PG Coaching, FMGE, USMLE

Sinusitis is one of the causes of orbital cellulitis which can progress as cavernous
sinus thrombosis.

Question: 20
Blepharophimosis syndrome [BPS] is generally associated with all of the
following except:

A Ectropion

B Distichiasis

C Epicanthus

D Telecanthus

14.31% People got this right

Explanation:

Correct Answer (B)

BPS is characterised by congenital ectropion of lower lid, Epicanthus Inversus [


the extra fold of skin arises from the lower lid] and Telecanthus [Far apart medial
canthus due to soft tissue problem , hence the IPD is normal] .

Distichiasis is a congenital anomaly where there is double row of eyelashes. It is


not a feature of this syndrome.

Question: 21
Which of the following clinical picture indicates a proliferative diabetic
retinopathy

https://emedicoz.com/testresult/683510s14693192 17/176
6/9/24, 12:45 PM Neet PG Preparation, Neet PG Coaching, FMGE, USMLE

A <p><img src="https://s3-
ap-south-
1.amazonaws.com/dams-apps-
production/ckfinder/files/image(4361).png"
style="height:181px; width:263px"
/></p>

B <p><img src="https://s3-
ap-south-
1.amazonaws.com/dams-apps-
production/ckfinder/files/image(4363).png"
style="height:257px; width:335px"
/></p>

C <p><img src="https://s3-
ap-south-
1.amazonaws.com/dams-apps-
production/ckfinder/files/image(4364).png"
style="height:131px; width:234px"
/></p>

D <p><img src="https://s3-
ap-south-
1.amazonaws.com/dams-apps-
production/ckfinder/files/image(4362).png"
style="height:184px; width:274px"
/></p>

38.1% People got this right

Explanation:

Correct Answer (B)

In b , we can see the NVD ie neovascualrisation at disc . hence it is a stage of


proliferative diabetic retinopathy.

a. Shows looping of veins which is a feature of moderate and severe NPDR

c. Mild NPDR with microaneurysms and haemorrhages.

d. Shows mild to moderate NPDR [ according to the ETDRS classification]


https://emedicoz.com/testresult/683510s14693192 18/176
6/9/24, 12:45 PM Neet PG Preparation, Neet PG Coaching, FMGE, USMLE

Question: 22
A tonic pupil, all are correct except:

A Occurs most commonly in


young women

B Will show segmental


vermiform iris contractions in
response to light

C May have light-near


dissociation

D Ultimately will become the


larger pupil

13.14% People got this right

Explanation:

Correct Answer (D)

In due course, a tonic pupil becomes constricted due to aberrant regeneration.

Question: 23
Carotid -cavernous fistula may be associated with all except:

A Diplopia

B Ocular ischemia

https://emedicoz.com/testresult/683510s14693192 19/176
6/9/24, 12:45 PM Neet PG Preparation, Neet PG Coaching, FMGE, USMLE

C Exophthalmos

D Enophthalmos

48.86% People got this right

Explanation:

Correct Answer (D)

In C-C fistula , there is a direct flow between the cavernous sinus and carotid
artery. These are high flow fistulas typically due to trauma. Diplopia is secondary
to decreased blood supply to the cranial nerves in the cavernous sinus. There is
arterialisation of episcleral veins . Other features are ocular ischemia , venous
stasis retinopathy and exophthalmos [pulsating proptosis with bruit and a thrill]

Question: 24
Amblyopia may be associated with all except:

A Amplification of the
crowding phenomenon

B Decreased contrast
sensitivity

C An afferent pupillary defect

D Abnormal spatial visual


processing

11.99% People got this right

Explanation:

Correct Answer (E)

https://emedicoz.com/testresult/683510s14693192 20/176
6/9/24, 12:45 PM Neet PG Preparation, Neet PG Coaching, FMGE, USMLE

In amblyopia there is decreased saccadic amplitudes.

Question: 25
All are true related to the condition given below except:

A Most frequently occur


medial to the infraorbital canal

B Associated with hypothesia


of cheek and upper gum suggest
infraorbital nerve damage

C Associated with positive


forced duction test always have
extraocular muscle entrapped in
the fracture

D Associated with medial wall


fracture presents with
subcutaneous emphysema

23.59% People got this right

Explanation:

Correct Answer (C)

https://emedicoz.com/testresult/683510s14693192 21/176
6/9/24, 12:45 PM Neet PG Preparation, Neet PG Coaching, FMGE, USMLE

A positive forced duction test and vertical diplopia may also occur due to facial
attachments of the inferior rectus muscle and also due to contusion injury to
inferior rectus.

Question: 26
CT scan of neck of a patient of carcinoma larynx has been found to have
the involvement of epiglottis, right ventricular band, right true vocal cord
and invading into pre-epiglottic space. What is the ideal treatment of this
patient?

A Radiotherapy

B Total laryngectomy +
radiotherapy

C Chemotherapy

D Chemoradiation

50.18% People got this right

Explanation:

Correct Answer (B)

Total laryngectomy + radiotherapy

Ref: Read the text below

Sol:

• This is stage T3 laryngeal carcinoma. The treatment of choice for T3 and T4


can- cer larynx is total laryngectomy followed by radiotherapy.

• However, certain cancer study groups recommend the use of con- current
chemoradiation with Cisplatin for T3 T4cancer larynx as an alternative to total
laryngectomy.

https://emedicoz.com/testresult/683510s14693192 22/176
6/9/24, 12:45 PM Neet PG Preparation, Neet PG Coaching, FMGE, USMLE

• Though, the 5 year survival rate with concurrent chemora- diation is lower as
compared to surgical management.

Question: 27
All of the following are true about pachydermia laryngitis except:

A Hoarseness of voice

B Biopsy shows acanthosis


and hyperkeratosis

C Premalignant condition

D Involves posterior part of


larynx

28.38% People got this right

Explanation:

Correct Answer (C)

Premalignant condition

Ref: Read the text below

Sol:

Reflux Laryngitis

• There is edema in both vocal cords and hypervascularity.

• Pachyderma, a condition of increasing mucosal tissue with swelling and


hypertrophy in the area of the arytenoids, is seen in the space posterior to the
vocal cords.

https://emedicoz.com/testresult/683510s14693192 23/176
6/9/24, 12:45 PM Neet PG Preparation, Neet PG Coaching, FMGE, USMLE

• It is not a premalignant lesion.

Question: 28
A mother brings her child saying he is not eating anything since last 8 hrs.
On history she confirm that child was playing with coin. On examination
vitals were stable and no dyspnea. Study the given Xrays and comment on
diagnosis:

A Foreign body in bronchus

B Foreign body in trachea

C Foreign body in larynx

D Foreign body in esophagus

61.12% People got this right

Explanation:

Correct Answer (D)

Foreign body in esophagus

Ref: Read the text below

Sol:

https://emedicoz.com/testresult/683510s14693192 24/176
6/9/24, 12:45 PM Neet PG Preparation, Neet PG Coaching, FMGE, USMLE

Question: 29
The picture given is an ossicular replacement prosthesis used in a situation
when there is erosion of Malleus, Incus and stapes Head. What is the name
of this prosthesis?

A Partial Ossicular
replacement prosthesis

B Total ossicular replacement


prosthesis

C Stapedial piston prosthesis

D Grommet

https://emedicoz.com/testresult/683510s14693192 25/176
6/9/24, 12:45 PM Neet PG Preparation, Neet PG Coaching, FMGE, USMLE

32.41% People got this right

Explanation:

Correct Answer (B)

Total ossicular replacement prosthesis

Ref: Read the text below

Sol:

PORP ( Partial ossicular replacement pros- thesis) is used in Type III


tympanoplasty situation when malleus and incus are ab- sent

TORP ( Total ossicular replacement pros- thesis) is used in Type IV tympanoplasty


situation when malleus , incus and sta- pes head are absent and only footplate is
there.

Question: 30
A congenitally deaf child with bilateral profound SNHL on radiological
evaluation has been found to have intact eighth nerve on both sides.
What is the best method of hearing rehabilitation in this case.

A Cochlear implant

B Digital Hearing aid

C Auditory brain stem


implant

D Bone anchored hearing aid

45.47% People got this right

Explanation:

Correct Answer (A)

https://emedicoz.com/testresult/683510s14693192 26/176
6/9/24, 12:45 PM Neet PG Preparation, Neet PG Coaching, FMGE, USMLE

Cochlear implant

Ref: Read the text below

Sol:

Cochlear Implant is a device that converts sound to electrical energy and directly
stimulates cochlear nerve fibres electrically (8th nerve). Hence, integrity of 8th
nerve is vital for this surgery

Indication

Bilateral profound or severe to profound hearing loss with no benefit seen with
hearing aid

Parts of cochlear implant:

A.External parts Microphone Speech processor TrAnsmitter Battery

B. Internal parts

Reciever stimulator (electrode ) Surgical technique..

A. The classical approach is posterior tympanotomy (facial recess approach)

B. Other surgical technique is Veria approach…. This is trAnscanal approach


Insertion of electrode through Cochleostomy/or through round window
membrane Where the electrodes are placed?

Scala tympani Types of implant:

Single channel (house- 3M device) Multiple channel (nucleus device)- 22 channel


—much better

Speech therapy is must after cochlear im- plantation

Mondini dysplasia of cochlea refers to the condition when there are one and half
turns of cochlea rather than two and half. There is absence of the apical
modiolus and interscalar septum, resulting in an incomplete partitioning of the
cochlea together with an enlarged vestibular aqueduct (EVA) and dilated
vestibule. This is not a contraindication of cochlear implantation.

https://emedicoz.com/testresult/683510s14693192 27/176
6/9/24, 12:45 PM Neet PG Preparation, Neet PG Coaching, FMGE, USMLE

Question: 31
50 yrs old female complaining of hearing loss, vertigo, pain and tinnitus.
Computed tomography reveals a hyperdense, pedunculated mass arising
from the tympanosquamous suture and lateral of the isthmus.
Histopathologically, mass is covered with periosteum and squamous
epithelium, and consist of lamalleted bone surrounding fibrovascular
channels with minimal osteocysts.What is the possible diagnosis of the
image shown below:

A Cholesteatoma of canal

B Keratosis obturans

C Osteoma

D Polyp arising from facial


nerve.

40.34% People got this right

Explanation:

Correct Answer (C)

Osteoma

Reference – Read the text below

https://emedicoz.com/testresult/683510s14693192 28/176
6/9/24, 12:45 PM Neet PG Preparation, Neet PG Coaching, FMGE, USMLE

Sol:

• Osteomas are sometimes referred to as exostoses of the ear canal. They


occur in two forms: 1) single cancellous osteomas or 2) multiple compact
osteomas.

• They are slow growing and never become malignant. They are three times
more common in males than in females and are often bilateral.

• The etiology is unknown, however, some researchers ascribe this condition to


swimming in cold water.

• Surgical excision is indicated when osteomas impede the natural outward


migration of earwax, thus causing wax impaction and hearing loss.

Question: 32
“Treacher Collins Syndrome” is characterized by all of the following
except :

A Autosomal dominant

B B/L facial agenesis

C High arched palate

D Bilateral sensorineural
deafness

22.76% People got this right

Explanation:

Correct Answer (D)

Bilateral sensorineural deafness

Sol:

https://emedicoz.com/testresult/683510s14693192 29/176
6/9/24, 12:45 PM Neet PG Preparation, Neet PG Coaching, FMGE, USMLE

TREACHER COLLINS SYNDROME (Mandibulofacial dysostosis)

o Autosomal dominant

o B/L facial agenesis

o Hypoplasia of the derivatives of the first and second branchial arches

o Facial Features

• Obliquely slanting palpebral fissure

• Depressed cheek bones

• Deformed pinna

• Receding chin

• Hypoplastic mandible

• Deficient ascending, ramus

• High arched palate

o Mental retardation

o B/L conductive deafness

Question: 33
Structure pointed in the image is known as

https://emedicoz.com/testresult/683510s14693192 30/176
6/9/24, 12:45 PM Neet PG Preparation, Neet PG Coaching, FMGE, USMLE

A Faucial tonsils

B Palatine tonsils

C Nasopharyngeal tonsils

D Lingual tonsils

54.46% People got this right

Explanation:

Correct Answer (C)

Nasopharyngeal tonsils

Ref: Read the text below

Sol:

• Nasopharyngeal tonsil,commonly called “adenoids”, is situated at the


junction of the roof and posterior wall of the nasopharynx

• Covering epithelium is of three types: Ciliated Pseudostratified columnar,


stratified squamous and transitional

• Blood supply

• Ascending palatine branch of facial

• Ascending pharyngeal branch of external carotid.

• Pharyngeal branch of the third part of maxillary artery.

• Ascending cervical branch of inferior thyroid artery of thyrocervical trunk

Question: 34
Choose the most inappropriate statement regarding investigation shown in
the given image
https://emedicoz.com/testresult/683510s14693192 31/176
6/9/24, 12:45 PM Neet PG Preparation, Neet PG Coaching, FMGE, USMLE

A cVEMP tests saccule and


inferior nerve

B cVEMP is an inhibitory
potential and an ipsilateral
response

C oVEMP is an excitatory
response that we record from the
extraocular muscles.

D oVEMP is primarily a utricle


and inferior nerve response

16.65% People got this right

Explanation:

Correct Answer (D)

oVEMP is primarily a utricle and inferior nerve response

Ref: Read the text below

Sol:

• We have to understand the difference between the cervical VEMP (cVEMP)


and the ocular VEMP (oVEMP).

https://emedicoz.com/testresult/683510s14693192 32/176
6/9/24, 12:45 PM Neet PG Preparation, Neet PG Coaching, FMGE, USMLE

• The cVEMP tests saccule and inferior nerve. It is recorded from the
sternocleidomastoid muscle (SCM). It is an inhibitory potential and an ipsilateral
response. When a muscle is flexed, there is a split second within that flex where
it releases. That provides the waveform or the response from the sound.

• The oVEMP is primarily a utricle and superior nerve response. A little part of
this oVEMP response comes from the saccule. This is an excitatory response that
we record from the extraocular muscles.

Question: 35
Mutations in which gap junction protein gene account for the highest
percentage of nonsyndromic congenital sensorineural hearing loss?

A Connexin 43

B Connexin 30

C Connexin 26

D Connexin 32

12.58% People got this right

Explanation:

Correct Answer (C)

Connexin 26

Sol

NONSYNDROMIC SENSORINEURAL HEARING LOSS

• SNHL in a child or young adult without known acquired cause and without
any associated unusual physical features is called nonsyndromic deafness.

https://emedicoz.com/testresult/683510s14693192 33/176
6/9/24, 12:45 PM Neet PG Preparation, Neet PG Coaching, FMGE, USMLE

• Most nonsyndromic forms of deafness are monogenic and rare, with the
exception of one form caused by inheritance of a homozygous autosomal
recessive mutant gene for the gap junction protein connexin 26.

Question: 36
A 62-year-old man comes to the physician complaining of double vision.
He first noticed mild difficulty focusing his eyes about 3 weeks ago and
his symptoms have progressively worsened. The patient's other medical
problems include metastatic prostate cancer On neurologic examination,
he is unable to adduct his left eye, and stimulation of the left cornea does
not elicit a corneal reflex. A lesion involving which of the following
anatomical structures is most likely responsible for this patient's
symptoms?

A Inferior orbital fissure

B Optic canal

C Superior orbital fissure

D Foramen rotundum

46.28% People got this right

Explanation:

Correct Answer (C)

Superior orbital fissure

Ref.: Read the text below

Sol :

This patient's diplopia is caused by his inability to adduct his left eye Eye
adduction depends on the oculomotor nerve (CN III) and the medial rectus
muscle. The oculomotor nerve originates in the oculomotor nucleus of the
https://emedicoz.com/testresult/683510s14693192 34/176
6/9/24, 12:45 PM Neet PG Preparation, Neet PG Coaching, FMGE, USMLE

midbrain, emerges from the anterior midbrain, and enters the orbit through the
superior orbital fissure. The patient also has loss of the corneal reflex on the left
side. The sensory limb of the corneal reflex is mediated by the nasociliary branch
of the first division of the trigeminal nerve (CN V,). The motor component of the
corneal reflex is carried primarily by the temporal branch of the facial nerve (CN
VII). Like the oculomotor nerve, the nasociliary nerve enters the orbit through
the superior orbital fissure. Thus, a lesion involving the superior orbital fissure
would cause the described deficits

The trochlear nerve (CN IV), abducens nerve (CN VI), and superior ophthalmic
vein also enter the orbit via the superior orbital fissure.

Question: 37
A 75-year-old patient presents to the dermatologist with a 4 cm brown
lesion on the right cheek (Figure). The patient states that the lesion has
been there for years but just recently began to change in appearance. She
has noticed cracking and oozing from the lesion over the past few months.
The lesion most likely originated from a cell derived from which germ cell
line

A Notochord

B Endoderm

C Neural crest cells

https://emedicoz.com/testresult/683510s14693192 35/176
6/9/24, 12:45 PM Neet PG Preparation, Neet PG Coaching, FMGE, USMLE

D Mesoderm

54.73% People got this right

Explanation:

Correct Answer (C)

Neural crest cells

Ref.: Read the text below

Sol :

This patient's lesion is most consistent with melanoma, which is derived from
melanocytes. Melanocytes are of neural crest cell origin.

Question: 38
A patient presents to your office with right leg numbness. When he walks,
you notice that he lifts his right foot higher than he does his left, and that
his right foot slaps to the ground with each step. On neurologic exam,
you also ascertain that he is unable to evert the right foot. Which of the
following nerves has most likely been injured?

A Tibial

B Common peroneal

C Superficial peroneal

D Sural

3.52% People got this right

Explanation:

Correct Answer (D)

https://emedicoz.com/testresult/683510s14693192 36/176
6/9/24, 12:45 PM Neet PG Preparation, Neet PG Coaching, FMGE, USMLE

Abducent nerve

Exp: Patients with common peroneal nerve damage present with an equinovarus
(plantarflexed and inverted) posture of the affected foot due to paralysis of the
peroneus longus and peroneus brevis muscles (mediate foot eversion), paralysis
of the tibialis anterior muscle (mediates dorsiflexion), and paralysis of the
extrinsic extensors of the toes. Injury to this nerve also causes loss of sensation
to the anterolateral leg.

The classic finding on gait exam in patients with common peroneal nerve injury
is "foot drop," where the affected leg is lifted high off of the ground while
walking due to an inability to dorsiflex the foot. The affected foot will also
classically slap to the ground with each step.

Question: 39
Safety muscle of larynx:

A Lateral cricoarytenoid

B Transverse artenoid

C Posterior cricoarytenoid

D Cricothyroids

54.04% People got this right

Explanation:

Correct Answer (C)

Posterior cricoarytenoid

Ref: Read the text below

https://emedicoz.com/testresult/683510s14693192 37/176
6/9/24, 12:45 PM Neet PG Preparation, Neet PG Coaching, FMGE, USMLE

Sol Because it is the only abductor of vocal cords

Question: 40
Smallest muscle in the body is:

A Interarytenoid

B Stapedius

C Corrugator supercilli

D Superior oblique

50.33% People got this right

Explanation:

Correct Answer (B)

Stapedius

Ref: Read the text below

Sol :

• The smallest skeletal muscle in the body is stapedius.

• The smallest muscle in the body is arrector pilorum, a smooth muscle in the
skin for erection of hair.

Question: 41
Which of the following is not included in "purposes of classification in
psychiatry"?

https://emedicoz.com/testresult/683510s14693192 38/176
6/9/24, 12:45 PM Neet PG Preparation, Neet PG Coaching, FMGE, USMLE

A Communication

B Control

C Comprehension

D Coordination

21.22% People got this right

Explanation:

Correct Answer (D)

Coordination

Its a recent exam question. Direct pick up from Kaplan comprehensive textbook
(not given in kaplan’s synopsis).. Purposes of classification are communication,
control and comprehension.

Question: 42
In which subtype of schizophrenia grimacing is a feature:

A Simple

B Hebephrenic

C Paranoid

D Catatonic

36.14% People got this right

Explanation:

https://emedicoz.com/testresult/683510s14693192 39/176
6/9/24, 12:45 PM Neet PG Preparation, Neet PG Coaching, FMGE, USMLE

Correct Answer (B)

Hebephrenic

Mirror gazing, silly smiling, giggling and grimacing are seen in hebephrenia

Question: 43
True regarding "neurotransmitters" in mood disorders are all except:

A Dopamine activity is
decreased in depression and
increased in mania

B cholinergic agonists can


exacerbate symptoms in
depression and can reduce
symptoms in mania

C Reduction in levels of
GABA has been observed in
plasma, CSF in c/o depression

D Glutamate activity is
decreased in depression.

13.24% People got this right

Explanation:

Correct Answer (D)

Glutamate activity is decreased in depression

“Dopamine activity may be reduced in depression and increased in mania.”

“Cholinergic Agonists can produce lethargy, anergia, and psychomotor


retardation in healthy subjects, can exacerbate symptoms in depression, and can
reduce symptoms in mania.

https://emedicoz.com/testresult/683510s14693192 40/176
6/9/24, 12:45 PM Neet PG Preparation, Neet PG Coaching, FMGE, USMLE

“Reductions of GABA have been observed in plasma, CSF, and brain GABA levels
in depression.” (These 3 statements are true as per pg 530, Kaplan synopsis 10th
edn)

Question: 44
Vasanthi 45 years was brought to casually with abnormal movement
which included persistent deviation of neck to right side, one day after
she was prescribed Haloperidol 5 mgs three times daily from the
psychiatry OPD. She also had an altercation with her husband recently,
which of the following is the most likely cause for her symptoms:

A Acute drug dystonia

B Conversion reaction

C Acute psychosis

D Cerebrovascular accident

42.34% People got this right

Explanation:

Correct Answer (A)

Acute drug dystonia

This clinical question has a catch, few of you are tempted to mark answer as
“Conversion reaction” only because of this statement of the question “She also
had an altercation with her husband recently”... It is not only d stressor that
decides the diagnosis of conversion disorder. Stress can precede any mental or
physical illness. The clinical picture is of acute onset extrapyramidal symptom,
drug dystonia, due to high dose of a typical neuroleptic.

https://emedicoz.com/testresult/683510s14693192 41/176
6/9/24, 12:45 PM Neet PG Preparation, Neet PG Coaching, FMGE, USMLE

Question: 45
A previously healthy 60-year-old man undergoes a corneal transplant.
Three months later, he is profoundly demented, demonstrates myoclonic
jerks on examination, and has an EEG that shows periodic bursts of
electrical activity superimposed on a slow background. Which of the
following is the most likely diagnosis?

A Pseudodementia

B Multi-infarct dementia

C Creutzfeldt- Jakob disease

D Epilepsy

53.6% People got this right

Explanation:

Correct Answer (C)

Creutzfeldt- Jakob disease

It is a neurodegenerative disease caused by a transmissible infectious agent, the


prion. Most cases are iatrogenic, following transplant of infected corneas or use
of contaminated neurosurgical instruments. Familial forms, following an
autosomal dominant pattern of inheritance,represent 5% to 15% of cases.
Patients show a very rapid cognitive deterioration, myoclonic jerks, rigidity, and
ataxia. Death follows within a year. An intermittent periodic burst pattern
(periodic complexes) is the characteristic EEG finding.

Epilepsy causes spike and wave patterns on EEG, and may cause postseizure
memory loss and disorientation (in generalized, tonic-clonic seizures) or a
depersonalization syndrome (in temporal lobe epilepsy or other focal seizure
disorder), but would not be expected to cause dementia, continuous myoclonic
jerks on examination, or an EEG that shows periodic bursts of electrical activity
superimposed on a slow background.

https://emedicoz.com/testresult/683510s14693192 42/176
6/9/24, 12:45 PM Neet PG Preparation, Neet PG Coaching, FMGE, USMLE

Pseudodementia is the term used for patients with major depression, who
exhibit impaired attention, perception, problem solving, or memory. The
cognitive decline is often more precipitous than for demented patients. Patient
history often reveals past major depressive episodes. Although the actual
memory impairment is modest in these patients, the subjective complaint is
great.

Question: 46
22-year old man has had a swollen wrist for several months. Diagnosis
please.

A Osteosarcoma

B Osteoclastoma

C Aneurysmal Bone Cyst

D Simple Bone cyst

35.68% People got this right

Explanation:

Correct Answer (B)

https://emedicoz.com/testresult/683510s14693192 43/176
6/9/24, 12:45 PM Neet PG Preparation, Neet PG Coaching, FMGE, USMLE

This lesion has a benign radiographic appearance. Note that, while the distal
radius is expanded, the cortex of the bone is intact. The margin of the tumor is
well demarcated. This is a giant cell tumor. These are usually benign although
approximately ten percent can be malignant. The key radiographic feature is the
extension of the lesion to the articular surface.

Question: 47
An 80-year-old female presents with hip pain and an inability to weight
bear. A radiograph was performed. What medication would you expect the
patient to be taking?

A Bisphosphonate

B Levothyroxine

C Methotrexate

D Prednisolone

32.92% People got this right

Explanation:

Correct Answer (A)

https://emedicoz.com/testresult/683510s14693192 44/176
6/9/24, 12:45 PM Neet PG Preparation, Neet PG Coaching, FMGE, USMLE

All the medications above can result in osteoporosis except for bisphosphonate,
which is a treatment for osteoporosis. However, long-term use of bisphosphates
can result in atypical proximal femoral fractures, for which this is a typical
example.

Question: 48
What Couinaud liver segment is highlighted?

A Segment II

B Segment III

C Segment IV

D Segment V

56.34% People got this right

Explanation:

Correct Answer (D)

The Couinaud classification is used to describe functional liver anatomy. It is the


preferred anatomy classification system as it divides the liver into eight
independent functional units (termed segments) rather than relying on the

https://emedicoz.com/testresult/683510s14693192 45/176
6/9/24, 12:45 PM Neet PG Preparation, Neet PG Coaching, FMGE, USMLE

traditional morphological description based on the external appearance of the


liver. The segments are numbered in roman numerals I to VIII.

Question: 49
Following admission 35 days earlier with acute pancreatitis, on a contrast-
enhanced CT study a collection with enhancing margins and
heterogenous content is present in the region of the pancreatic head and
neck. This should be reported as a:

A Acute necrotic collection

B Acute peripancreatic fluid


collection

C Pancreatic pseudocyst

D Walled off necrosis

21.29% People got this right

Explanation:

Correct Answer (D)

In acute pancreatitis, collections with enhancing margins are encapsulated and


are generally seen after 4 weeks. The presence of non-liquid elements
differentiates between a pancreatic pseudocyst (see in interstitial oedematous
pancreatitis) and walled-off necrosis (seen in necrotising pancreatitis). According
to the Revised Atlanta Classification, the term pancreatic abscess should be
abandoned.

Question: 50

https://emedicoz.com/testresult/683510s14693192 46/176
6/9/24, 12:45 PM Neet PG Preparation, Neet PG Coaching, FMGE, USMLE

Which site is typically not involved by lymphadenopathy in abdominal


tuberculosis?

A Peripancreatic

B Retroperitoneal

C Mesenteric

D Porta hepatis

22.3% People got this right

Explanation:

Correct Answer (B)

Tuberculous lymph nodes are typically enlarged, of low attentuation due to


central necrosis, and have enhancing margins. They tend to be clustered. The
retroperitoneal compartment is less frequently involved than the porta hepatis,
peripancreatic and mesenteric regions.

Question: 51
A 37-year-old man comes to the emergency department because of
increasing pain and tenderness in his right forearm. During a bar brawl 6
days earlier, he sustained a 4—cm laceration through the skin and
subcutaneous tissue of his forearm. Treatment at the time of injury
included cleaning and dressing the wound. Physical examination shows
erythema surrounding the wound site and expression of yellow pus when
pressure is applied adjacent to the wound. Which of the following
molecules is most likely responsible for causing accumulation of pus over
this patient's wound?

A C3a

https://emedicoz.com/testresult/683510s14693192 47/176
6/9/24, 12:45 PM Neet PG Preparation, Neet PG Coaching, FMGE, USMLE

B lL-3

C lL-8

D lL-10

18.88% People got this right

Explanation:

Correct Answer (C)

IL-8

Pus consists of a thin, protein-rich fluid, known as liquor puris, and dead
leukocytes, primarily neutrophils.
During infection, macrophages and surrounding endothelial cells release
cytokines such as interleukin-8 (IL-8) that trigger neutrophils to enter the site of
infection via chemotaxis.
lL-B also induces phagocytosis in neutrophils once they have arrived

Question: 52
A 23-year-old woman presented to the clinic for her first prenatal
appointment with fatigue and pain in the perineum for the past 8 days.
The past medical history is benign and she claimed to have only had
unprotected intercourse with her husband. She had a documented
allergic reaction to Amoxicillin 2 years ago. The vaginal speculum exam
revealed a clean, ulcerated genital lesion, which was tender and non-
exudative. No lymphadenopathy was detected. A rapid plasma regain
(RPR) test revealed a titer of 1:64 and the fluorescent treponemal
antibody absorption (FTA-abs) test was positive. What is the next best
step in the management of this patient?

A Parenteral ceftriaxone. 1 g
x 10 days

https://emedicoz.com/testresult/683510s14693192 48/176
6/9/24, 12:45 PM Neet PG Preparation, Neet PG Coaching, FMGE, USMLE

B Penicillin desensitization.
then intramuscular benzathine
penicillin, G 2.4 million units

C Oral tetracycline, 500 mg 4


times daily x 1week

D Doxycycline, 100 mg twice


daily x 14 days

38.32% People got this right

Explanation:

Correct Answer (B)

Penicillin desensitization. then intramuscular benzathine penicillin, G 2.4 million


units

This patient has primary syphilis, which is consistent with the symptoms and
laboratory results.
Specifically, the genital ulcer, positive and positive RPR with a high titer support
the diagnosis.
Parenteral benzylpenicillin is the only recommended treatment for pregnant
women.
If a patient has a well-documented allergic reaction to penicillin, as in this
patient, desensitization to penicillin is recommended until a full dose is
tolerated.
Gravidas should receive penicillin according to the stage of syphilis, although
some specialists suggest that gravidas who present in the third trimester with
early syphilis should receive 2 doses rather than 1.

Question: 53
A 5-year-old child is brought to a pediatric clinic by his mother for a rash
that started a few days ago. She adds that her son also has had a fever and
sore throat since last week. His immunizations are up—to—date. On
examination. a rash is present over the trunk and upper extremities that
feels like sandpaper to the touch. The oropharyngeal examination is

https://emedicoz.com/testresult/683510s14693192 49/176
6/9/24, 12:45 PM Neet PG Preparation, Neet PG Coaching, FMGE, USMLE

suggestive of exudative pharyngitis with a white coat over the tongue as


shown in the image. The physician swabs the throat and uses the swab in a
rapid antigen detection test kit. He also sends the sample for a
microbiological culture. The physician then recommends empiric antibiotic
therapy and tells the mother that if the boy is left untreated, the likelihood
of developing a complication later in life is very high. Which of the
following best explains the mechanism underlying the development of the
possible complication the physician is talking about?

A Antigenic shift

B Bacterial tissue invasion

C Molecular mimicry

D Toxin-mediated cellular
damage

44.08% People got this right

Explanation:

Correct Answer (C)

Molecular mimicry

This patient's clinical presentation is highly suggestive of scarlet fever which is


caused by Streptococcus pyogenes. a gram-positive bacteria that is catalase
negative.

https://emedicoz.com/testresult/683510s14693192 50/176
6/9/24, 12:45 PM Neet PG Preparation, Neet PG Coaching, FMGE, USMLE

The complication that the physician is talking about is rheumatic fever, the
pathogenesis of which arises from the host immune responses.
The M protein found on the surface of the group A streptococci Streptococcus
pyogenes is recognized as an antigen by the immune system.
M protein triggers an immune reaction, leading to antibody formation.
Interestingly. the amino acid sequence in this bacterial protein is similar to the
amino acid sequence found in the sarcolemmal protein of human cardiac
myocytes.
Thus. the anti-M protein antibodies cross-react with cardiac tissue, damaging
the heart and causing other clinical manifestations leading to rheumatic fever.
This process is called antigenic mimicry. in which the bacterial proteins ‘mimic’
the proteins found in host tissues. Antigenic mimicry is a microbial mechanism
to escape host immunologic defenses.

Question: 54
A 24—year-old woman presents with fever, abdominal pain, and bloody
bowel movements. She says her symptoms onset 2 days ago and have
not improved. She describes the abdominal pain as moderate, cramping
in character, and poorly localized. 1 week ago, she says she was on a
camping trip with her friends and had barbecued chicken which she
thought tasted strange. The patient denies any chills, hemoptysis,
hematochezia, or similar symptoms in the past. Her vital signs include:
pulse 87/min and temperature 318°C. Physical examination is significant
for moderate tenderness to palpation in the periumbilical region with no
rebound or guarding. Stool is guaiac positive. Microscopy revealed Gull
wing shape bacteria with darting motility. Which of the following is a
complication associated with this patient‘s most likely diagnosis?

A Appendicitis

B Toxic megacolon

C Guillain-Barré syndrome

D Hemolytic uremic
syndrome

https://emedicoz.com/testresult/683510s14693192 51/176
6/9/24, 12:45 PM Neet PG Preparation, Neet PG Coaching, FMGE, USMLE

37.66% People got this right

Explanation:

Correct Answer (C)

Guillain-Barré syndrome

This patient presents with acute dysentery bloody Diarrhea following the
ingestion of chicken most likely contaminated with Campylobacter jejuni.
Guillain-Barré syndrome frequently develops in patients with a recent history of
C. jejuni gasteroenteritis.
C. jejuni is one of the bacterial agents responsible for inflammatory enterocolitis,
an infection of the colon/large intestines, causing mucus and bloody diarrhea.
It is transmitted via undercooked meat [especially poultry), contaminated water,
and unpasteurized milk.
C jejuni has an incubation period of 2-40 days after inoculation, following by
diarrhea fever, cramps, and vomiting. Although it causes inflammation, diarrhea
is not always bloody, although a stool guaiac is typically positive.
Stool examination will usually show increased polymorphonuclear leukocytes
and lactoferrin.
C. jejuni may cause Guillain-Barré syndrome, an acute rapidly demyelinating
neuropathy, immunoglobulin A (lgA) nephropathy, erythema nodosum.
hemolytic anemia, and reactive arthritis.
GBS usually occurs a few weeks after the infection and is considered a
neurological emergency due to the risk of respiratory failure.
The prognosis is good although 15—20% may have complications.

Question: 55
A 26-year-old previously healthy man comes to the emergency department
due to fever, malaise, anorexia, and painless swelling of the left eye. His
symptoms began after a month-long anthropology trip to rural village.
During his stay, he lived in an adobe house with a thatched roof and
unnetted windows and doors. Temperature is 38.30 C. Physical examination
shows left-sided periorbital swelling and conjunctival erythema with no
discharge. The remainder of the physical examination, including right eye
examination, shows no abnormalities. Giemsa-stained peripheral blood
smear is shown in the image. This patient is at greatest risk for which of the
following complications if his condition is left untreated?

https://emedicoz.com/testresult/683510s14693192 52/176
6/9/24, 12:45 PM Neet PG Preparation, Neet PG Coaching, FMGE, USMLE

A Ascending aortic aneurysm

B Calcified brain lesions

C Cardiac arrhythmias

D Large joint arthritis

18.49% People got this right

Explanation:

Correct Answer (C)

Cardiac arrhythmias

Chagas disease is a vector-home protozoan illness endemic to most Latin


American countries. Risk is greatest in those with long-term domestic exposure
to the Triatomine bug, which nests in thatched roofing and the cracks of adobe
or unfinished brick. Transmission occurs when the bug feeds on human blood
during the night and defecates the underlying pathogen, Trypanosoma cruzi,
into the bite wound.
Chagas disease is divided into 2 phases : The acute phase (8-12 weeks after
transmission) is characterized by circulating trypomastigotes that are detectable
on blood microscopy. Most patients are asymptomatic, but a minority develop
nonspecific symptoms (eg, fever, malaise, anorexia) and infiammation, swelling
at the site of inoculation (eg, eye swelling following conjunctival inoculation).
The chronic phase begins when the immune system eliminates circulating
parasitemia, leading to resolution of acute symptoms. Patients subsequently
develop a prolonged asymptomatic phase ("indeterminate form"), whereby

https://emedicoz.com/testresult/683510s14693192 53/176
6/9/24, 12:45 PM Neet PG Preparation, Neet PG Coaching, FMGE, USMLE

serology is positive for T.cruzi, but there are no symptoms, signs, detectable
parasitemia, or indications of end-organ damage. After 1-3 decades, a minority
of those with asymptomatic chronic infection develop end-organ disease of the
heart or gastrointestinal system.
Chronic Chagas cardiomyopathy is the most common complication of Chagas
disease. it is thought be caused by chronic, low-grade, parasite-mediated
myocarditis, which leads to the progressive destruction of cardiac fibers and
subsequent cardiac fibrosis. Common complications include biventricular heart
failure, cardiac arrhythmias (particularly ventricular arrhythmias), and ventricular
aneurysm with intracardiac thrombus (leads to thromboembolic disease -stroke)

Question: 56
During the course of a week at an overnight summer. camp. 4 children
aged 7-9 are sent to the camp health center. They each have fever, cough,
congestion, sore throat, and red eyes. Physical examination of the
children shows bilateral conjunctival injection and an erythematous
oropharynx. Auscultation of one child also reveals crackles in the left
lower lung field. All the children's symptoms improve over 7 days with
supportive care. Which of the following is the most likely cause of the
outbreak?

A Adenovirus

B Coxsackie virus

C Influenza virus

D Norovirus

37.83% People got this right

Explanation:

Correct Answer (A)

Adenovirus

https://emedicoz.com/testresult/683510s14693192 54/176
6/9/24, 12:45 PM Neet PG Preparation, Neet PG Coaching, FMGE, USMLE

Adenovirus infection is a common, usually self-limiting, disease caused by a


double-stranded DNA genome that is transmitted via direct contact, fecal-oral
route, or respiratory droplets.
Adenovirus infection occurs year-round and In outbreaks In crowded quarters
(eg, day care centers, military barracks)
Viral replication in epithelial cells can give rise to varying manifestations of
disease based on serotype, with upper respiratory involvement as the most
common. Pharyngoconjunctival fever is characterized by the acute onset of
fever, cough, congestion, pharyngitis, and conjunctivitis. Physical examination
usually reveals an erythematous oropharynx with bilateral, follicular conjunctival
injection and serous discharge.
Pneumonia occurs in a small percentage of patients, such as the child with focal
crackles.

Question: 57
Find the incorrect statement regarding the image given below.

A First Plasmid DNA vaccine


against COVID 19.

B Second indigenous vaccine


produced in India

C Permitted by DCGI by
routine approval protocols.

https://emedicoz.com/testresult/683510s14693192 55/176
6/9/24, 12:45 PM Neet PG Preparation, Neet PG Coaching, FMGE, USMLE

D Needle free administration

14.07% People got this right

Explanation:

Correct Answer (C)

Permitted by DCGI by routine approval protocols.

All the Vaccines against COVID 19 in India are approved by Emergency Use
Authorization by DGCI.

Zycov D is the first DNA vaccine used against SARS CoV 2 and three doses on
0,28 and 56 days.

Question: 58
A 47-year-old man initially comes to his primary care physician with
persistent fever, night sweats, and fatigue. Thorough evaluation yields a
diagnosis of chronic myeloid leukemia. While undergoing treatment for his
malignancy, the patient comes to the oncologist complaining of headaches,
scant nasal discharge, and a problem with his left eye. Physical examination
reveals tenderness over the paranasal sinuses in addition to left-sided
orbital swelling and cellulitis. Mild proptosis and ptosis of the left eye are
also present. Biopsy of his sinus mucosa is shown in the image. Choose the
correct etiological agent

https://emedicoz.com/testresult/683510s14693192 56/176
6/9/24, 12:45 PM Neet PG Preparation, Neet PG Coaching, FMGE, USMLE

A Aspergillus species

B Rhizopus species

C Histoplasma capsulatum

D Cryptococcus neoformans

19.1% People got this right

Explanation:

Correct Answer (A)

Aspergillus species

This patient's symptoms are consistent with fungal rhinosinusitis.


The biopsy image shows fungal hyphae branching at acute angles (thick arrows)
with septations (thin arrows), which is characteristic of Aspergillus.
This fungus is widely distributed in the environment and commonly grows on
decaying vegetables. It is monomorphic, existing only in mold form (ie,
multicellular hyphae).

Question: 59
A 26-year-old man presents to his primary care physician with abdominal
cramps and diarrhea of 1-week duration. He states he has been feeling
more tired than normal, feels bloated and is passing an unusual amount
of gas. He mentions that he went camping in the Village side Mountains
about 2 weeks ago and drank water from the stream at his campsite. At
the time of examination, his temperature is 37.1°C . Abdominal
examination reveals diffuse tenderness. Laboratory results are pending.
Which of the following is the most likely causative agent?

A Entamoeba histolytica

https://emedicoz.com/testresult/683510s14693192 57/176
6/9/24, 12:45 PM Neet PG Preparation, Neet PG Coaching, FMGE, USMLE

B Giardia lamblia

C Necator americanus

D Norwalk virus

35.8% People got this right

Explanation:

Correct Answer (B)

Giardia lamblia

This is a case of acute giardiasis.


Classic findings of acute Giardia infections include foul-smelling watery diarrhea,
abdominal cramps, bloating, flatulence, steatorrhea, nausea, anorexia, and
malaise.
Infections are transmitted via contaminated food and water and the organisms
can survive in freshwater, especially mountain streams, such as described in this
case. The organism is also transmitted via the fecal–oral route and anal–oral
sexual contact.

Question: 60
A 20-year-old college student presents to the university student health
service. The patient reports to the health care provider that he has
developed a fever, malaise, and cough productive of scant, whitish
sputum over the last few days. Rales are heard in the right posterior lung
field. Chest X-ray reveals a patchy consolidation in the right lower lobe.
Cold agglutinins are elevated. The patient is started on a 3-day course of
antibiotics and quickly recovers. Which of the following antibiotics is
ineffective against this pathogen?

A Azithromycin

B Ceftriaxone

https://emedicoz.com/testresult/683510s14693192 58/176
6/9/24, 12:45 PM Neet PG Preparation, Neet PG Coaching, FMGE, USMLE

C Doxycycline

D Erythromycin

22.1% People got this right

Explanation:

Correct Answer (B)

Ceftriaxone

It s a case of Atypical pneumonia caused by Mycoplasma pneumoniae which is


is notable for not having a cell wall.

Ceftriaxone, a third-generation cephalosporin, is a β-lactam antibiotic. It’s


mechanism of action involves binding to PBPs, which prevent transpeptidation
of cell wall precursors thus preventing cell wall synthesis.

Ceftriaxone and all other β-lactams do not have any antimicrobial activity
against M. pneumoniae, since it does not have a cell wall.

Question: 61
Which of the following is not an established complication of partial
gastrectomy?

A Constipation

B Osteomalacia

C Iron deficiency anemia

D Megaloblastic anemia

27.59% People got this right

https://emedicoz.com/testresult/683510s14693192 59/176
6/9/24, 12:45 PM Neet PG Preparation, Neet PG Coaching, FMGE, USMLE

Explanation:

Correct Answer (A) Patient develops diarrhoea, but not constipation. Patient can
become anemic by two mechanism, iron deficiency by failure of iron absorption
and megaloblastic anemia due to vit B 12 deficiency. This occurs due to loss of
intrinsic factor secretion by parietal cells. Osteomalacia occurs as result of
inefficient calcium and Vit. D absorption.

Question: 62
Regarding base of skull fracture, which of the following would not be an
expected finding?

A Battle’s sign

B CSF rhinorrhoea

C Hemotympanum

D Paralysis of
sternocleidomastoid

39.12% People got this right

Explanation:

Correct Answer (D) Clinical evidence of a skull base fracture may include Battle’s
sign and ‘racoon’ or ‘panda’ eyes (bilateral periorbital bruising).
Haemotympanum, or overt bleeding from the ear if the tympanic membrane has
ruptured, and CSF rhinnorrhoea or otorrhoea are also highly suggestive of a
fracture of the base of the skull.

Question: 63
A 72-year-old man presents with right flank pain and fever. A contrast-
enhanced CT scan is shown in Figure. The most
likely diagnosis is:

https://emedicoz.com/testresult/683510s14693192 60/176
6/9/24, 12:45 PM Neet PG Preparation, Neet PG Coaching, FMGE, USMLE

A Acute right renal


obstruction.

B Xanthogranulomatous
pyelonephritis.

C Cellulitis in right flank.

D Right perinephric abscess.

24.84% People got this right

Explanation:

Correct Answer (D) Right perinephric abscess. The CT scan is obtained in the late
arterial to nephrographic phase of the examination (the aorta is still opacified
with contrast agent), before the excretion of the contrast agent. Thus option b is
incorrect. There are multiple calculi in the right kidney, which is small and
atrophic, indicating a chronic process (thus option a is incorrect). There is
thickening of the perinephric fascia, and gas bubbles are seen in the posterior
paranephric space, extending to the right flank. In addition, there are fluid
collections in the posterior paranephric space and in the soft tissues of the right
flank, making option d the most likely diagnosis. Xanthogranulomatous
pyelonephritis is a chronic inflammatory condition associated with staghorn
calculi. The affected kidney is usually enlarged rather than shrunken, as is the
case here.

https://emedicoz.com/testresult/683510s14693192 61/176
6/9/24, 12:45 PM Neet PG Preparation, Neet PG Coaching, FMGE, USMLE

Question: 64
A 60-year-old man presents with a 6-mm basal cell carcinoma on the tip
of his nose. He is scheduled to undergo excision of the tumor in the
operating room with repair of the defect using skin and subcutaneous
tissue from his earlobe. Which of the following terms most appropriately
describes this form of reconstructive surgery?

A Split-thickness graft

B Full-thickness graft

C Composite graft

D Pedicle flap

15.02% People got this right

Explanation:

Correct Answer (C) A composite tissue graft contains tissue in addition to the
epidermis and dermis. Depending on the donor site, it can contain subcutaneous
fat, cartilage, and even muscle. This makes it ideal for deeper defects in which a
partial-thickness or full-thickness skin graft would give a poor cosmetic
outcome. A flap is a vascularized piece of tissue that is mobilized from its
original site to another location. A free tissue flap involves severing the original
blood supply with anastomosis to blood vessels in the recipient site. The
remaining defect in this patient is too small to warrant the need for a flap.

Question: 65
A 15 years old girl, known case of thalassemia major, with history of
multiple blood transfusions, presents with sudden abdominal pain. ON
examination she has pallor and vitals are unstable. How would you
proceed?

https://emedicoz.com/testresult/683510s14693192 62/176
6/9/24, 12:45 PM Neet PG Preparation, Neet PG Coaching, FMGE, USMLE

A Administer
hydroxychloroquine

B Stabilization and
splenectomy

C Blood transfusion only

D None of the above

4.86% People got this right

Explanation:

Correct Answer (B) Acute Splenic sequestration and hypersplenism are life
threatening disorders in children with thalassemia and sickle cell disease. These
conditions result in raid splenic enlargement and pain abdomen. Resuscitation
with hydration and blood transfusion may be followed by splenectomy in these
patients.

Question: 66
Which of the following statements regarding ductal adenocarcinoma of
the pancreas is/are correct?

A Fifty percent of pancreatic


adenocarcinomas involve the
gland diffusely at the time of
diagnosis

B For ductal
adenocarcinomas, tumors of the
body and tail are usually larger at
the time of diagnosis than those
arising in the head of the gland

https://emedicoz.com/testresult/683510s14693192 63/176
6/9/24, 12:45 PM Neet PG Preparation, Neet PG Coaching, FMGE, USMLE

C Pancreatic adenocarcinoma
occur with equal frequency within
the head, body, and tail of the
gland

D Acinar cell carcinoma of


pancreas is the most common
variety

24.3% People got this right

Explanation:

Correct Answer (B) Ductal adenocarcinoma is the most common malignancy of


pancreas with 60 – 70 percent of tumors arising in the head of the gland, 15
percent in the body and 10 percent in the tail. The remainder are diffuse. Acinar
cell carcinoma is extremely rare. Tumors from the body and tail are larger at
diagnosis, because obstructive jaundice is not seen at an early stage.

Question: 67
Biliary strictures developing after laparoscopic cholecystectomy usually
occurs in which part of the bile duct?

A Upper part near hepatic


duct

B Middle part below cystic


duct insertion

C Lower part near ampulla

D All sites with equal


frequency

11.87% People got this right

https://emedicoz.com/testresult/683510s14693192 64/176
6/9/24, 12:45 PM Neet PG Preparation, Neet PG Coaching, FMGE, USMLE

Explanation:

Correct Answer (A) Approximately 80% of benign strictures occur following


injury during a cholecystectomy. The incidence rate of major bile duct injury in
0.2% - 0.3% after open cholecystectomy and 0.4-0.6% after laparoscopic
cholecystectomy. 75% injuries go unnoticed. Presentation may be s much as 5
years later. The length of strictures are usually short and occur more commonly
in the common hepatic duct.

Question: 68
A 29-year-old man sustained a gunshot wound to the right upper
quadrant. He is taken to the operating room and, after management of a
liver injury, is found to have a complete transection of the common bile
duct with significant tissue loss. Which of the following is the optimal
surgical management of this patient’s injury?

A Choledochoduodenostomy

B Loop
choledochojejunostomy

C Primary end-to-end
anastomosis of the transected
bile duct

D Roux-en-Y
choledochojejunostomy

40.83% People got this right

Explanation:

Correct Answer (D) Complete transection of the common bile duct can be
handled in many ways. If the patient is unstable and time is limited, simply
placing a T tube in either end of the open common bile duct and staging the
repair is the treatment of choice. In a stable patient with a transected bile duct

https://emedicoz.com/testresult/683510s14693192 65/176
6/9/24, 12:45 PM Neet PG Preparation, Neet PG Coaching, FMGE, USMLE

and a loss of tissue, a biliary enteric bypass is preferred. This can be


accomplished by Roux-en-Y choledochojejunostomy or cholecystojejunostomy.

Question: 69
A 55-year-old woman who has end-stage liver disease is referred to a
hepatologist for evaluation. Which of the following would prevent her
from being a transplantation candidate?

A Use of alcohol 3 months


ago

B Two 2-cm hepatocellular


carcinomas (HCCs) in the right
lobe of the liver

C A 4-cm hepatocellular
carcinoma in the right lobe of the
liver

D Development of
hepatorenal syndrome requiring
hemodialysis

13.75% People got this right

Explanation:

Correct Answer (A) The more common contraindications to liver transplantation


are ongoing or recent substance abuse, presence of active sepsis, current
extrahepatic malignancy, poor cardiac or pulmonary function, and patients with
hepatocellular carcinoma with metastatic disease, obvious vascular invasion, or
significant tumor burden. Patients with hepatocellular carcinoma who would not
tolerate resection because of portal hypertension and uncompensated liver
disease can be successfully treated with liver transplantation. The best
candidates are patients with a single lesion less than 5 cm in size or no more
than 3 lesions, none of which are greater than 3 cm in size. The presence of

https://emedicoz.com/testresult/683510s14693192 66/176
6/9/24, 12:45 PM Neet PG Preparation, Neet PG Coaching, FMGE, USMLE

hepatorenal syndrome is an indication, not a contraindication, to liver


transplantation.

Question: 70
Active bleeding in angiodysplasia patients can be treated by following
methods except?

A Intra-arterial vasopressin

B Sclerotherapy

C Selective gel foam


embolization

D Radiation

41.81% People got this right

Explanation:

Correct Answer (D) Angiodysplasias of the intestine, also referred to as


arteriovenous malformations, are distinct from haemangiomas and true
congenital arteriovenous malformations. They are thought to be acquired
degenerative lesions secondary to progressive dilation of normal blood vessels
within the submucosa of the intestine. In acutely bleeding patients, they have
been successfully treated with intra-arterial vasopressin, selective gel foam
embolization, endoscopic electrocoagulation, or injection with sclerosing agents.
If these measures fail or bleeding recurs and the lesion has been localized,
segmental resection, most commonly right colectomy, is effective.

Question: 71
The best reason for using the technique shown in picture is:

https://emedicoz.com/testresult/683510s14693192 67/176
6/9/24, 12:45 PM Neet PG Preparation, Neet PG Coaching, FMGE, USMLE

A It Has A Higher Sterilization


Rate Than Standard Vasectomy
With incision.

B Patients are rendered


sterile in less time.

C It is easier to learn than the


standard technique.

D It results in a lower rate of


complications, including
hematoma and infection.

45.59% People got this right

Explanation:

Correct Answer (D) The technique shown is no scalpel vasectomy(NSV) ,It results
in a lower rate of complications, including hematoma and infection. This method
eliminates the scalpel incision, results in fewer hematomas and infections, and
leaves a much smaller wound than conventional methods of accessing the vas
deferens for vasectomy.

Question: 72
What type of malignancy is most common in patients with achalasia?

https://emedicoz.com/testresult/683510s14693192 68/176
6/9/24, 12:45 PM Neet PG Preparation, Neet PG Coaching, FMGE, USMLE

A Squamous cell carcinoma

B Adenocarcinoma

C Transitional cell malignancy

D None of the above

36.83% People got this right

Explanation:

Correct Answer (A) Achalasia is also known to be a premalignant condition of


the esophagus. During a 20-year period, a patient will have up to an 8% chance
for development of carcinoma. Squamous cell carcinoma is the most common
type identified and is thought to be the result of long-standing retained
undigested fermenting food in the body of the esophagus, causing mucosal
irritation.

Question: 73
A 22-year-old woman has a known family history of breast cancer in her
first-degree relatives. She undergoes genetic testing and is found to be a
BRCA1 mutation carrier. She does not currently desire bilateral
prophylactic mastectomy. Which of the following is the next best option
to manage her risk for breast cancer?

A Mammography every 6
months starting at age 25

B Mammography every 6
months starting at age 35

C Mammography every 12
months starting at age 20

https://emedicoz.com/testresult/683510s14693192 69/176
6/9/24, 12:45 PM Neet PG Preparation, Neet PG Coaching, FMGE, USMLE

D Tamoxifen for
chemoprevention

11.45% People got this right

Explanation:

Correct Answer (D) Both BRCA1 and BRCA2 are associated with an increased
cumulative risk of both breast and ovarian cancer. Prophylactic bilateral
mastectomy and reconstruction is recommended for BRCA mutation carriers. If
prophylactic mastectomy is not performed, intensive surveillance for breast
cancer with biannual clinical examinations and annual mammograms starting at
age 30 is recommended. Tamoxifen is not routinely indicated for all BRCA1
carriers since most breast cancers in BRCA1 mutation carriers are estrogen
receptor-negative. BRCA2 breast cancers are more likely to be estrogen
receptor-positive. Approximately 10% of women under the age of 40 who
develop breast cancer have a mutation in BRCA1 or 2. BRCA1 is associated with
an increased risk of colon cancer and prostate cancer in males. BRCA2 is
associated with an increased risk of gallbladder, bile duct, and pancreatic
cancers as well as gastric cancer, malignant melanoma, and in men, prostate
cancer.

Question: 74
Which of the following is the most common variety of esophageal
diverticulum?

A Zenker

B Mid esophageal

C Traction

D Epiphrenic

49.84% People got this right

Explanation:
https://emedicoz.com/testresult/683510s14693192 70/176
6/9/24, 12:45 PM Neet PG Preparation, Neet PG Coaching, FMGE, USMLE

Correct Answer (A) The pharyngoesophageal diverticulum (Zenker diverticulum)


is the most common esophageal diverticulum found today. It is usually
manifested in older patients in the seventh decade of life and has been
postulated to be a result of loss of tissue elasticity and muscle tone with age. It is
specifically found herniating from Killian’s triangle, between the oblique fibers of
the thyropharyngeus muscle and the horizontal fibers of the cricopharyngeus
muscle.

Question: 75
Which of the following is removed in the surgery of following neck
swelling?

A Antenor Jugular vein

B External Jugular vein

C Hyoid bone

D Superior laryngeal cartilage

48.25% People got this right

Explanation:

Correct Answer (C) Surgical management of a thyroglossal duct cyst requires


removal of the cyst, the tract, and the central portion of the hyoid bone (Sistrunk
procedure), as well as a portion of the tongue base up to the foramen cecum.

https://emedicoz.com/testresult/683510s14693192 71/176
6/9/24, 12:45 PM Neet PG Preparation, Neet PG Coaching, FMGE, USMLE

Before excision of a thyroglossal duct cyst, an imaging study such as ultrasound


is performed to identify if normal thyroid tissue exists in the lower neck, and lab
assay is performed to assess if the patient is euthyroid.

Question: 76
Ivor Lewis approach for esophageal resection in carcinoma esophagus is?

A Laparotomy with cervical


approach

B Left thoracotomy and


laparotomy

C Transhiatal cervical
anastomosis

D Right thoracotomy and


laparotomy

23.61% People got this right

Explanation:

Correct Answer (D)

Ivor lewis procedure usually denotes a transthoracic esophagectomy with


intrathoracic anastomosis.

https://emedicoz.com/testresult/683510s14693192 72/176
6/9/24, 12:45 PM Neet PG Preparation, Neet PG Coaching, FMGE, USMLE

Question: 77
Which of the following statements about chylolymphatic cyst mesenteric
cyst is not true?

A Treatment involves
resection of bowel

B More common in small


bowel

C More common in men

D Aspiration alone has high


rates of recurrence

23.71% People got this right

Explanation:

Correct Answer (A)

Enucleation of the cyst at laparotomy is curative and can generally be


accomplished because the mesenteric blood vessels and intestinal wall are
usually not adherent to the cyst wall. Internal drainage of the cyst into the
peritoneal cavity has also been successfully used in the treatment of very large
cysts. Aspiration alone has a high rate of cyst recurrence. In those cases in which

https://emedicoz.com/testresult/683510s14693192 73/176
6/9/24, 12:45 PM Neet PG Preparation, Neet PG Coaching, FMGE, USMLE

the cyst is not completely excised, the contents of the cyst and the internal
architecture of the cyst wall must be carefully inspected and the cyst wall
examined histologically to rule out a non-neoplastic cause. 2 common types of
mesenteric cysts are;

Question: 78
Which of the following patients should undergo elective repair of an
asymptomatic abdominal aortic aneurysm?

A A man with a 4.5-cm


aneurysm

B A woman with a 5.0-cm


aneurysm

C A mun with an aneurysm


that has grown 0.5 cm in the last
year

D A woman with an
aneurysm that has grown 0.5 cm
in the last year

26.86% People got this right

Explanation:

https://emedicoz.com/testresult/683510s14693192 74/176
6/9/24, 12:45 PM Neet PG Preparation, Neet PG Coaching, FMGE, USMLE

Correct Answer (B)

The rupture risk is quite low for aneurysms < 5.5 cm and begins to rise
exponentially thereafter. This size can serve as an appropriate threshold for
recommending elective repair provided one's surgical mortality is below 5%. For
each size strata, however, women appear to be at higher risk for rupture than
men, and a lower threshold of 4.5 to 5.0 cm maybe reasonable in good-risk
patients Although data are less compelling, a pattern of rapid expansion of >0.5
cm within 6 months can be considered a relative indication for elective repair.

Question: 79
Which of the following is true regarding hepatic adenomas?

A Acute episodes of bleeding


are best managed by surgical
resection.

B They tend to appear “hot”


on a sulfur colloid liver scan.

C Rapid contrast
enhancement on CT distinguishes
them from FNH.

D A 5.5-cm lesion should be


resected even if it is
asymptomatic.

25.64% People got this right

Explanation:

Correct Answer (D) Differentiating FNH and hepatic adenoma is not always
straightforward. Both may show contrast enhancement in the arterial phase of a
CT scan, so this does not help to differentiate them. FNH characteristically
demonstrates a central scar. Adenomas may demonstrate increased fat signal on
MRI compared with FNH. When CT and MRI are unable to distinguish adenoma

https://emedicoz.com/testresult/683510s14693192 75/176
6/9/24, 12:45 PM Neet PG Preparation, Neet PG Coaching, FMGE, USMLE

from FNH, a sulfur colloid scan may be beneficial because adenomas will appear
“cold” and FNHs “hot” because of the presence of Kupffer cells. Radiofrequency
ablation is another potential option in managing hepatic adenomas, especially
when multiple adenomas are present, or the patient is not a candidate for a
major liver resection.

Question: 80
Ten Horn sign is seen in?

A Portal hypertension

B Acute cholecystitis

C Acute appendicitis

D Acute pancreatitis

22.56% People got this right

Explanation:

Correct Answer (C)

In case of suspected acute appendicitis in males, gentle traction of the


right permatic cord causes pain in the right iliac fossa. Few other named signs
are

https://emedicoz.com/testresult/683510s14693192 76/176
6/9/24, 12:45 PM Neet PG Preparation, Neet PG Coaching, FMGE, USMLE

Question: 81
The preferred definitive treatment for recurrent acute pancreatitis due to
pancreas divisum is:

A Lateral
pancreaticojejunostomy (Puestow
procedure)

B Pancreaticoduodenectomy
(Whipple procedure)

C Minor papilla
sphincterotomy

D Major papilla
sphincterotomy and pancreatic
ductal septotomy

11.16% People got this right

Explanation:

Correct Answer (C) Pancreas divisum can lead to recurrent episodes of acute
pancreatitis as well as chronic pancreatitis with intractable pain. Unlike other
forms of chronic pancreatitis, however, marked dilation of the dorsal duct is
unusual. As such, surgical decompressive procedures are not successful. For

https://emedicoz.com/testresult/683510s14693192 77/176
6/9/24, 12:45 PM Neet PG Preparation, Neet PG Coaching, FMGE, USMLE

patients with recurrent attacks of acute pancreatitis, the best option is


sphincterotomy of the minor papilla because the duct of Santorini is providing
the primary drainage to the pancreas. A decreased rate of acute pancreatitis in
24 patients after minor papilla sphincterotomy and dorsal duct stenting is seen.
The complication rate was lower with sphincterotomy than with stent insertion.
Major papilla sphincterotomy would not likely be helpful because it drains a
minority of the pancreas in pancreatic divisum.

Question: 82
An elderly female develops incisional hernia after abdominal hysterectomy
done 5 yrs back. Image is given. As per the EHS classification this hernia is

A M1

B M3

C M2

D M5

22.56% People got this right

Explanation:

Correct Answer (D)

M5

https://emedicoz.com/testresult/683510s14693192 78/176
6/9/24, 12:45 PM Neet PG Preparation, Neet PG Coaching, FMGE, USMLE

Question: 83
Which of the following is true regarding the principles of the operative
management of the small bowel in Crohn disease?

A The optimal margin is at


least 4 cm beyond grossly visible
disease.

B Frozen section should be


obtained to confirm the absence
of active disease in at least one
margin.

C A 3-cm strictured segment


of duodenum is best managed by
resection.

D A 10-cm strictured
segment of jejunum can be
managed by a Heineke-Mikulicz
strictureplasty rather than by
resection.

14.43% People got this right

https://emedicoz.com/testresult/683510s14693192 79/176
6/9/24, 12:45 PM Neet PG Preparation, Neet PG Coaching, FMGE, USMLE

Explanation:

Correct Answer (D)

Approximately three-fourths of patients with Crohn disease will eventually


require surgery.

Indications for surgery include failure of medical management, intestinal


obstruction, fistula, abscess, bleeding, and perforation. In children, growth
retardation is another indication. Because patients with Crohn disease will often
require repeat operations, it is important to avoid unnecessary resection of small
bowel because this puts the patient at risk of short bowel syndrome. As such,
several principles of surgical management should be followed. Surgical resection
should be limited to the segment of bowel that is causing the complication.
Other areas of active disease should be left alone provided they are not causing
obvious complications. Resection margins of 2 cm beyond grossly visible disease
are recommended . Resection margins have not been shown to affect
recurrence. The presence of microscopic disease in the resection margin also
does not adversely affect outcome or recurrence. Thus, frozen section is
unnecessary

When the indication for surgery is SBO, strictureplasty has been shown to be
equally effective as resection for jejunal and ileal disease while sparing bowel
length. Two types of strictureplasty are recommended: the Heineke-Mikulicz
pyloroplasty (for strictures <12 cm in length) and the Finney pyloroplasty (for
strictures ≤25 cm in length) . A potential drawback of these techniques is that
they may potentially leave an undetected malignancy behind. Thus, during the
course of a strictureplasty, biopsy specimens of any intraluminal ulcerations
should be taken. Duodenal Crohn disease is much less common, and thus
guidelines are less clear. However, current recommendations are to perform a
bypass of duodenal strictures, such as with a gastrojejunostomy and
duodenojejunostomy, depending on the location. Duodenal resection is not
recommended . Duodenal strictureplasty has been rarely reported. For colon
disease, resection is recommended, again limiting resection to the diseased
segment causing symptoms. In a metaanalysis, 90% of recurrences occurred at
nonstrictureplasty sites.

https://emedicoz.com/testresult/683510s14693192 80/176
6/9/24, 12:45 PM Neet PG Preparation, Neet PG Coaching, FMGE, USMLE

Question: 84
A 20-year-old morbidly obese man sustains a GSW to the abdomen. His
blood pressure is 110/70 mm Hg and his heart rate is 100 beats per
minute. At surgery, he is found to have a blast injury to the sigmoid colon
involving 75% of the circumference of the bowel, with a moderate
amount of fecal contamination. Which of the following is the best option?

A Sigmoid colectomy with


primary anastomosis with a
diverting ileostomy

B Primary repair of the


sigmoid colon

C Sigmoid colectomy with


primary anastomosis

D Primary repair of the


sigmoid colon with exteriorization
of the repair

5.25% People got this right

Explanation:

Correct Answer (C)

Increasingly, colon injuries are being treated with either primary repair, if
feasible, or resection with a primary anastomosis . This approach applies to both
right- and left-sided colon injuries. Primary repair is used when less than 50% of
the circumference of the bowel is involved, whereas resection is recommended
for larger wounds. Once a resection is performed, a decision must be made as to
whether to perform a primary reanastomosis or a colostomy. The
primary contraindication to attempting a primary reanastomosis is
hemodynamic instability. In these situations, damage control surgery should be
performed and the decision to reanastamose or create a colostomy can be made
at a subsequent operation when the patient has stabilized and been fully
resuscitated. Factors associated with intra-abdominal complications in patients
https://emedicoz.com/testresult/683510s14693192 81/176
6/9/24, 12:45 PM Neet PG Preparation, Neet PG Coaching, FMGE, USMLE

with severe colon injuries undergoing resection include severe fecal


contamination, transfusion of 4 or more units of blood in the first 24 hours, and
administration of single-agent antibiotics. The use of vasopressors at the time of
repair may also be associated with anastomotic leaks, whereas the method of
performing the anastomosis (handsewn versus stapled) has not been shown to
effect leak rates. Another important consideration is obesity. Morbid obesity
makes the creation of a stoma difficult, predisposes the stoma to the
development of ischemia, and, if this occurs, increases the risk of the
development of a necrotizing soft-tissue infection. It also makes the subsequent
colostomy takedown more challenging. As such, strong consideration should be
given in obese patients to a primary reanastomosis.

Question: 85
Type IIIA in Nyhus classification of hernia:

A Direct inguinal hernia

B Indirect inguinal hernia

C Femoral hernia

D Umbilical hernia

18.34% People got this right

Explanation:

Correct Answer (A)

https://emedicoz.com/testresult/683510s14693192 82/176
6/9/24, 12:45 PM Neet PG Preparation, Neet PG Coaching, FMGE, USMLE

Question: 86
A 25 year old Basket ball player while playing developed palpitations
,dyspnea and syncope. On ECHO –Systolic Anterior Movement of mitral
leaflet with massive LVH seen. Which of the following statement is not
correct about this patient

A Pulsus Bisferiens seen

B ECG will show giant


inverted T waves

C Murmur will decrease on


Valsalva maneuver

D Beta blocker will be useful

31.09% People got this right

Explanation:

Correct Answer (C) This is a case of HOCM.The clinical features include pulsus
Bisferiens and ejection systolic murmur which increases on standing and with
Valsalva effect.ECG will show LVH and giant inverted T waves.Initial drug of
choice is betablocker.

https://emedicoz.com/testresult/683510s14693192 83/176
6/9/24, 12:45 PM Neet PG Preparation, Neet PG Coaching, FMGE, USMLE

Question: 87
What is the mechanism of hyperandrogenism seen in polycystic ovary
syndrome?

A Genetic mutation in 3β-


hydroxysteroid dehydrogenase

B Higher FSH compared with


LH synthesis by the pituitary
gland

C Reduced aromatisation of
androgens by theca cells

D Reduction in circulating
SHBG

26.4% People got this right

Explanation:

Correct Answer (D) Hyperinsulinemia leads to a reduction in SHBG and resultant


increase in metabolically active free androgens. Mutations in 3β-hydroxysteroid
dehydrogenase cause a virilising form of congenital adrenal hyperplasia.

Question: 88
Austin Flint Murmur is

A Mid diastolic murmur seen


in Severe Mitral Stenosis

B Early diastolic murmur seen


in Severe Aortic Regurgitation

https://emedicoz.com/testresult/683510s14693192 84/176
6/9/24, 12:45 PM Neet PG Preparation, Neet PG Coaching, FMGE, USMLE

C Mid diastolic murmur seen


in Severe Aortic Regurgitation

D Ejection systolic murmur


seen in Severe Aortic Stenosis

33.68% People got this right

Explanation:

Correct Answer (C) Austin flint murmur is due to aortic regurgitant blood striking
mitral valve producing mid diastolic murmur heard at apex.

Question: 89
Pel-Ebstein fever is seen in:

A Hodgkin’s disease

B Malaria

C Kalaazar

D Typhoid fever

50.77% People got this right

Explanation:

Correct Answer (A) Pel–Ebstein fever is seen in Hodgkin's lymphoma in which the
patient experiences fevers which cyclically increase over 1week followed by
plateau for 1 week then decrease over an average period of one or two weeks.

Question: 90

https://emedicoz.com/testresult/683510s14693192 85/176
6/9/24, 12:45 PM Neet PG Preparation, Neet PG Coaching, FMGE, USMLE

A 21-year young man presents in emergency with high grade fever with
inflammation of knee and ankle joints which is fleeting in nature. There is
history of occasional palpitations. There is no history of antecedent sore
throat. On examination, splenomegaly is present. The ECG shows
prolongation of PR interval. What is the most likely diagnosis in India?

A Still disease

B Enteric fever

C Rheumatic fever

D Reiter’s syndrome

16.26% People got this right

Explanation:

Correct Answer (D) In reactive arthritis joint involvement is usually asymmetric


and additive, with involvement of new joints occurring over a few days to 1–2
weeks. The joints of the lower extremities, especially the knee, ankle, subtalar,
metatarsophalangeal, and toe interphalangeal joints, are most commonly
involved. The characteristic skin lesions, keratoderma blennorrhagica, consist of
vesicles and/or pustules that become hyperkeratotic. Other manifestations of
ReA include cardiac conduction defects, aortic insufficiency, central or peripheral
nervous system lesions, and pleuropulmonary infiltrates.

Question: 91
The ‘Y’ descent in JVP waveform represents:

A Atrial relaxation

B Atrial systole

https://emedicoz.com/testresult/683510s14693192 86/176
6/9/24, 12:45 PM Neet PG Preparation, Neet PG Coaching, FMGE, USMLE

C Atrial emptying

D Apical displacement of
tricuspid valve

46.72% People got this right

Explanation:

Correct Answer (C) The y decent in JVP is due to atrial blood passively going into
ventricle during early filling phase of ventricles.

Question: 92
A patient with a diagnosis of scleroderma who has diffuse cutaneous
involvement presents with malignant hypertension, oliguria, edema,
hemolytic anemia, and renal failure. You make a diagnosis of scleroderma
renal crisis. Which of the following is the recommended treatment?

A Captopril

B Carvedilol

C Clonidine

D Diltiazem

29.94% People got this right

Explanation:

Correct Answer (A) The prognosis for patients with scleroderma renal disease is
poor. In scleroderma renal crisis patients, prompt treatment with an ACE
inhibitor may reverse acute renal failure.

https://emedicoz.com/testresult/683510s14693192 87/176
6/9/24, 12:45 PM Neet PG Preparation, Neet PG Coaching, FMGE, USMLE

Question: 93
Which of the following malignancies are patients with rheumatoid
arthritis specifically at higher risk for?

A Colon cancer

B Lung cancer

C Lymphoma

D Melanoma

35.12% People got this right

Explanation:

Correct Answer (C) There two- to fourfold increased risk of lymphoma in RA


patients compared with the general population. The most common
histopathologic type of lymphoma is a diffuse large B-cell lymphoma. The risk of
developing lymphoma increases if the patient has high levels of disease activity
or Felty syndrome.

Question: 94
A 55 year old woman with type 2 diabetes complaining of genital thrush.
She had been started on a new oral anti diabetes drug 4 months earlier.
Which of the following drugs is most likely to be responsible for her
presentation?

A DPP-4 inhibitor

B Glucosidase inhibitor

C PPARγ agonist

https://emedicoz.com/testresult/683510s14693192 88/176
6/9/24, 12:45 PM Neet PG Preparation, Neet PG Coaching, FMGE, USMLE

D SGLT2 inhibitor

36.97% People got this right

Explanation:

Correct Answer (D) Inhibitor of the sodium and glucose co-transporter 2 (SGLT2)
in the kidney, exerts its glycaemic effect by increasing the amount of glucose in
the urine. This can result in fungal infection of genital area.

Question: 95
A 65-year-old woman with diabetes, hypertension, coronary artery
disease, gastroesophageal reflux disease, and ongoing use of alcohol and
tobacco, presents with several months of increasing midsternal chest
discomfort predominantly when swallowing solid food. She has not noted
blood in her stool or melena. There is no weight loss. Laboratory studies
are normal. What is the most likely cause of her dysphagia?

A Esophageal cancer

B Peptic esophageal stricture

C Achalasia

D Zenker diverticulum

27.99% People got this right

Explanation:

Correct Answer (B)

Peptic strictures due to chronic, persistent acid reflux


Esophageal cancer is likely to have anemia and weight loss.
Achalasia often regurgitate undigested food
Zenker diverticulum is an outpouching in the posterior wall of the hypopharynx,
which allows food retention, causing halitosis, recurrent aspiration, and
https://emedicoz.com/testresult/683510s14693192 89/176
6/9/24, 12:45 PM Neet PG Preparation, Neet PG Coaching, FMGE, USMLE

pneumonia.

Question: 96
A 50-year-old man complains of abdominal pain, and wrist and knee pain
over the past several months. He has history of diabetes. On examination,
the patient has diffuse hyperpigmentation and a palpable liver 4 cm
below costal margin. Swelling of the wrists and metacarpophalangeal
joints is also noted. What is the next line of investigation to diagnose this
patient’s condition?

A RA factor and anti CCP ab

B Hemoglobin A1C

C Iron studies

D Alpha-1-antitrypsin level

26.03% People got this right

Explanation:

Correct Answer (C)

Clinical features are suggestive of Hemochromatosis

liver is enlarged,
skin pigmentation
Diabetes secondary to direct damage to the pancreas by iron deposition
Arthropathy
Initial screening involves transferrin saturation
Transferrin saturation of over 40% in women or 50% in men or a ferritin over 300
would be consistent with the diagnosis and would suggest the need for
confirmatory genetic testing.

https://emedicoz.com/testresult/683510s14693192 90/176
6/9/24, 12:45 PM Neet PG Preparation, Neet PG Coaching, FMGE, USMLE

Question: 97
A patient presented with jaundice but no vomiting, pain abdomen or
fever. Lab test- CBC: normal, Total bilirubin: 2.7 mg/dL, Direct bilirubin:
0.3 mg/dL, AST: normal, ALT: normal, Alkaline phosphatase: normal, Urine
bilirubin: negative. What is the most likely explanation for this man’s
jaundice?

A Glucuronyl transferase
deficiency.

B Warm antibodies directed


against red blood cell antigens.

C Common bile duct


obstruction.

D Hepatocellular injury

29.57% People got this right

Explanation:

Correct Answer (A) This patient has isolated hyperbilirubinemia (hepatocellular


enzymes and alkaline phosphatase are normal) and hyperbilirubinemia is
unconjugated (direct bilirubin is normal and indirect bilirubin is high). Causes of
unconjugated isolated hyperbilirubinemia-hemolysis or defective conjugation of
bilirubin related to deficiencies in glucuronyl transferase (Gilbert syndrome and
Crigler Najjar syndrome).

Question: 98
A 76-year-old man presents to the emergency room. He had influenza
and now complains of diffuse muscle pain and weakness. His medical
history is remarkable for hypercholesterolemia for which he takes
atorvastatin. Physical examination reveals blood pressure of 130/90 with
no orthostatic change. The only other finding is diffuse muscle
tenderness. Laboratory data include BUN: 30 mg/dL Creatinine: 6 mg/dL
https://emedicoz.com/testresult/683510s14693192 91/176
6/9/24, 12:45 PM Neet PG Preparation, Neet PG Coaching, FMGE, USMLE

K: 6.0 mEq/L Uric acid: 18 mg/dL Ca: 6.5 mg/dL Po4 : 7.5 mg/dL, Urine:
large blood, 2+ protein. Microscopic study shows muddy brown casts and
0 to 2 RBC/hpf. Which of the following is the most likely diagnosis?

A Nonsteroidal anti-
inflammatory drug-induced acute
kidney injury (AKI)

B Volume depletion

C Rhabdomyolysis-induced
acute kidney injury

D Urinary tract obstruction

51.53% People got this right

Explanation:

Correct Answer (C)

Rhabdomyolysis-induced AKI is characterized by hyperkalemia,


hyperphosphatemia, and hyperuricemia and hypocalcemia.

The dipstick registers red blood cells, hemoglobin (eg, from intravascular
hemolysis), and myoglobin as “blood.”

Causes:Trauma, medications (especially statins), infectious processes (influenza,


sepsis), and extreme muscular exertion (seizures, exertional heat stroke).

Question: 99
A 20-year-old man presents with obtundation. His laboratory values are
as follows: Na: 138 mEq/LK: 4.2 mEq/L HCO3 : 5 mEq/L Cl: 104 mEq/L
Creatinine: 1.0 mg/dL BUN: 14 mg/dL Ca: 10 mg/dL Arterial blood gas on
room air: Po2 96, Pco2 15, pH 7.02 Blood glucose: 90 mg/dL Urinalysis:

https://emedicoz.com/testresult/683510s14693192 92/176
6/9/24, 12:45 PM Neet PG Preparation, Neet PG Coaching, FMGE, USMLE

Normal, without blood, protein, or crystals. Which of the following is the


most likely acid-base disorder?

A Pure normal anion-gap


metabolic acidosis

B Respiratory acidosis

C Pure high anion-gap


metabolic acidosis

D Combined high anion-gap


metabolic acidosis and respiratory
alkalosis

22.59% People got this right

Explanation:

Correct Answer (C) The first step in analysing an acid-base disturbance is simply
to look at the pH. This patient has an acidosis. Then look at the HCO3 and the
Pco2 to determine the primary disturbance; that is, is it a metabolic acidosis or a
respiratory acidosis? The serum HCO3 has decreased from 24 to 5 mEq/L, so this
must be a metabolic acidosis. The Pco2 is below the normal value of 40 mm, so
this cannot be a respiratory acidosis (the Pco2 would be above 40 in a
respiratory acidosis). The first two steps are straightforward and unambiguous.
The third (and most difficult) step is to assess the compensatory response. This
patient has a metabolic acidosis, so you need to assess the respiratory
compensation. That is to say, has the Pco2 decreased appropriately to
compensate for the metabolic acidosis? The normal compensatory response in
metabolic acidosis is for the Pco2 to decrease by 1.25 mm Hg for each 1-mEq
decrease in HCO3 . This patient’s 19 mEq/L drop in bicarbonate is matched by a
25-mm drop in the Pco2 . Hence, this is a compensated metabolic acidosis.
Another method of assessing compensation in a metabolic acidosis is to use the
Winters formula, which says that the appropriate Pco2 equals 1.5 (HCO3 ) + 8.
This would give an appropriate Pco2 of 15.5, very close to the measured Pco2 .
Again, the compensatory response is appropriate for the degree of acidosis; the
patient does not have a respiratory acid-base disorder. The fourth step is to
calculate the anion gap. The normal anion gap (Na- [Cl + HCO3 ]) is 8 to 10

https://emedicoz.com/testresult/683510s14693192 93/176
6/9/24, 12:45 PM Neet PG Preparation, Neet PG Coaching, FMGE, USMLE

mEq/L; in this case the value is 29 mEq/L. Therefore, this is a wide anion-gap
metabolic acidosis with appropriate respiratory compensation.

Question: 100
A patient presented with acute renal failure. Lab values are- serum
creatinine 3.5 mg/dL, serum sodium is 140 mEq/L, serum K 4.6 mEq/L,
and BUN 50 mg/dL. Urinalysis shows granular casts, RBC -nil, proteins nil.
Urine sodium is 50 mEq/L and urine creatinine is 35 mg/dL. What is the
most likely cause of this patient’s acute kidney injury?

A cute interstitial nephritis

B Acute glomerulonephritis

C Acute tubular necrosis

D Prerenal azotemia

28.3% People got this right

Explanation:

Correct Answer (C) FeNa = Urine sodium x plasma creatinine x 100/plasma


sodium x urine creatinine is 3.5 which does in favour of intrinsic AKI. Granular
(sometimes called “muddy brown”) casts in the urine points toward acute
tubular necrosis (ATN). Interstitial nephritis more commonly occurs in patients
following exposure to certain medications and typically is associated with white
blood cells (especially eosinophils) in the urine. Glomerulonephritis will have red
cell casts on the urinalysis.

Question: 101
A 50-year-old diabetic admitted with altered sensorium has these
Laboratory values- Na: 122 mEq/L K: 3.5 mEq/L Cl: 100 mEq/L HCO3 : 5
mEq/L BUN: 40 m/dL Cr: 1.8 mg/dL Glucose: 800 mg/dL Serum

https://emedicoz.com/testresult/683510s14693192 94/176
6/9/24, 12:45 PM Neet PG Preparation, Neet PG Coaching, FMGE, USMLE

osmolality: Elevated, Serum ketones: Negative. What is the most likely


cause of this patient’s hyponatremia?

A SIADH

B Adrenal crisis

C Hyperglycemia-related
hyponatremia

D Hyperglycemia

4.25% People got this right

Explanation:

Correct Answer (D) This patient is in a hyperosmolar state due to his


uncontrolled type 2 diabetes and recent alcohol ingestion. Severe
hyperglycaemia pulls free water from the cell, leaving the plasma hyponatremic.
SIADH and adrenal insufficiency can cause hyponatremia but would be
associated with low serum osmolality.

Question: 102
A 35-year-old woman diagnosed with Mycoplasma pneumoniae. Her
hemoglobin is 9.0 g/dL and MCV is 110. Which of the following is the best
next diagnostic test?

A Serum protein
electrophoresis

B Flow cytometry

C Peripheral blood smear

https://emedicoz.com/testresult/683510s14693192 95/176
6/9/24, 12:45 PM Neet PG Preparation, Neet PG Coaching, FMGE, USMLE

D Bone marrow biopsy

28.84% People got this right

Explanation:

Correct Answer (C)

Macrocytic anemia in the setting of Mycoplasma infections can be due to


autoimmune hemolytic anemia. Examination of the peripheral blood smear is
the first step in evaluation of hemolytic anemia. The young red cells (which
would show up as reticulocytes when properly stained) are much larger than
mature RBCs, accounting for the macrocytosis (the MCV can be as high as 140
with vigorous Reticulocytosis). The presence of micro spherocytes suggests
immune-mediated hemolysis, while the presence of fragmented RBCs or
schistocytes suggests a mechanical cause of hemolysis, as seen in the
microangiopathic hemolytic anaemias.

Serum protein electrophoresis is useful to diagnose multiple myeloma, which is


rarely associated with hemolysis, but this would not be the best initial test; the
anemia in multiple myeloma is normocytic.

Flow cytometry can detect surface proteins such as CD55, CD59 on granulocytes
and red blood cells in paroxysmal nocturnal haemoglobinuria (a rare cause of
hemolysis), but again is not the best first test.

Glucose-6-PD levels might be useful once hemolytic anemia is established by a


peripheral smear and negative Coombs test.

Bone marrow biopsy would show erythroid hyperplasia, but is usually not
required to diagnose hemolytic anemia.

Question: 103
Which of the following options are not matched correctly

A Diabetes- longest nerve


fibers are affected first

https://emedicoz.com/testresult/683510s14693192 96/176
6/9/24, 12:45 PM Neet PG Preparation, Neet PG Coaching, FMGE, USMLE

B Vitamin B12 deficiency-


posterior column more affected
than spinothalamic tract

C Multiple sclerosis-
hyporeflexia

D Hypothryoidism- delayed
relaxation stretch reflexes

22.27% People got this right

Explanation:

Correct Answer (C)

In metabolic neuropathies like diabetes, the longest nerve fibres are affected
first, leading to the stocking-glove pattern of sensory loss.
In vitamin B12 deficiency, posterior column function would be affected out of
proportion to small pain and temperature fibres.
Multiple sclerosis is an upper motor neuron disease that will cause
hyperreflexia.
Hypothyroidism causes delayed relaxation phase of muscle stretch reflexes

Question: 104
A 68-year-old man with a history of hypertension and coronary artery
disease presents with right-sided weakness, sensory loss, and an expressive
aphasia. Symptoms began 6 hours prior to arrival in the ED. Neuroimaging
studies are shown in the following figure. In the emergency department the
patient’s blood pressure is persistently 180/96. Which of the following is the
best next step in management of this patient’s blood pressure?

https://emedicoz.com/testresult/683510s14693192 97/176
6/9/24, 12:45 PM Neet PG Preparation, Neet PG Coaching, FMGE, USMLE

A Administer IV nitroprusside

B Administer clonidine 0.1


mg po until the blood pressure
drops below 140/90

C Observe the blood


pressure

D Administer IV labetalol

11.33% People got this right

Explanation:

Correct Answer (C) Since cerebral autoregulation is disrupted in acute stroke, a


drop in blood pressure can decrease perfusion and worsen the so-called
ischemic penumbra. Generally, blood pressure elevation up to 220/120 is
tolerated. If the patient were a candidate for thrombolytic therapy, it would be
acceptable to lower the BP to less than 185/110 with labetalol or nicardipine.
Since this patient’s symptoms began 6 hours ago, however, he is not a candidate
for rtPA. Some stroke specialists recommend more aggressive blood pressure
control in acute intracranial haemorrhage, but this patient has an ischemic (not
haemorrhagic) stroke. Mannitol is of minimal benefit in cerebral edema
associated with acute stroke.

Question: 105

https://emedicoz.com/testresult/683510s14693192 98/176
6/9/24, 12:45 PM Neet PG Preparation, Neet PG Coaching, FMGE, USMLE

Which of the following pulmonary function set is characteristic of COPD?

A FEV1 / FVC < 70% ; FEV1 ↓ ;


FVC ↓ ; TLC ↑ DLCO Normal

B FEV1 / FVC < 70% ; FEV1 ↑ ;


FVC ↓ ; TLC ↓ DLCO ↓

C FEV1 / FVC < 70% ; FEV1 ↓ ;


FVC ↑ ; TLC ↑ DLCO ↑

D FEV1 / FVC < 70% ; FEV1 ↓ ;


FVC ↓ ; TLC ↓ DLCO ↓

21.1% People got this right

Explanation:

Correct Answer (A)

The PFT measures in pulmonary conditions are as follows-

Question: 106
The parameters used in modified Child-Pugh classification for staging
cirrhosis are:

https://emedicoz.com/testresult/683510s14693192 99/176
6/9/24, 12:45 PM Neet PG Preparation, Neet PG Coaching, FMGE, USMLE

A Serum bilirubin, serum


albumin only

B Serum albumin, serum


bilirubin, prothrombin time only

C Serum albumin, serum


bilirubin, prothrombin time,
ascities only

D Serum albumin, Serum


bilirubin, prothrombin time,
ascities, hepatic encephalopathy

51.18% People got this right

Explanation:

Correct Answer (D)

Child Pugh classification is done for severity assessment of cirrhosis.

Question: 107
Which of the following is the most important complication in a patient
with coeliac disease, who was previously doing well on a gluten-free diet
and is now not responding to gluten restriction?

https://emedicoz.com/testresult/683510s14693192 100/176
6/9/24, 12:45 PM Neet PG Preparation, Neet PG Coaching, FMGE, USMLE

A Intestinal infection

B Intestinal lymphoma

C Resistant coeliac disease

D Type 2 Diabetes mellitus

38.68% People got this right

Explanation:

Correct Answer (B) Small intestinal lymphoma is the most important


complication of celiac sprue and clinically is suggested by failure to respond to
gluten free diet after responding earlier.

Question: 108
Which of the following is/are neurological manifestation(s) of vitamin B12
deficiency?

A Poor memory

B Optic atrophy

C Personality change

D All

52.97% People got this right

Explanation:

Correct Answer (D) Vitamin B12 is needed for the myelination of the central
nervous system. Its deficiency may cause a bilateral peripheral neuropathy or
degeneration (demyelination) of the cervical and thoracic posterior and lateral
https://emedicoz.com/testresult/683510s14693192 101/176
6/9/24, 12:45 PM Neet PG Preparation, Neet PG Coaching, FMGE, USMLE

(pyramidal) tracts of the spinal cord and, less frequently, of the cranial nerves
and of the white matter of the brain. Optic atrophy and cerebral symptoms
including dementia, depression, psychotic symptoms, and cognitive impairment
may be prominent. There may also be anosmia and loss of taste. MRI may show
the “spongy” degeneration of the cord. Replenishment of body stores should be
complete with six 1000-μg IM injections of hydroxocobalamin given at 3- to 7-
day intervals. For maintenance therapy, 1000 μg hydroxocobalamin IM once
every 3 months.

Question: 109
A 28-year old man weighs 80 kg and has a height of 150 cm. Based on his
body mass index, he will be categorized as:

A Normal

B Underweight

C Overweight

D Obese

46.62% People got this right

Explanation:

Correct Answer (D)

The patient has BMI of 35.5 which is obesity class II.

https://emedicoz.com/testresult/683510s14693192 102/176
6/9/24, 12:45 PM Neet PG Preparation, Neet PG Coaching, FMGE, USMLE

Question: 110
The following ECG rhythm can be seen due to

A Digoxin

B Hyperkalemia

C Hypocalcemia

D Hyponatremia

35.78% People got this right

Explanation:

Correct Answer (A) This ecg rhythm shows every alternate beat is due to
premature ventricular ectopic known as bigeminy rhythm.This is seen due to
https://emedicoz.com/testresult/683510s14693192 103/176
6/9/24, 12:45 PM Neet PG Preparation, Neet PG Coaching, FMGE, USMLE

digoxin toxicity.Hyperkalemia causes tall T waves with narrow ST


segment.Hypocalcemia causes prolong QT interval.Hyponatremia typically
doesn’t cause ECG changes.

Question: 111
A 60-year-old woman with metastatic small cell lung cancer presents to the
Emergency with nausea, vomiting, and tachycardia. She is diagnosed with
Addison disease. Which of the following set of findings are most likely in
the patient?

A A

B B

C C

D D

43.69% People got this right

Explanation:

Correct Answer (C) The patient has primary adrenal insufficiency due to bilateral
adrenal destruction from the metastatic lung cancer. The deficiency of cortisol
results in hypoglycemia. Aldosterone (mineralocorticoid) acts on the collecting
duct of the kidney to increase sodium reabsorption and potassium secretion.
Mineralocorticoid deficiency results in hyponatremia and hyperkalemia.
https://emedicoz.com/testresult/683510s14693192 104/176
6/9/24, 12:45 PM Neet PG Preparation, Neet PG Coaching, FMGE, USMLE

Hyponatremia and the subsequent volume depletion results in a decreased


blood pressure.

Question: 112
A hypertensive patient develops chronic renal failure from progressive
nephrosclerosis. Which of the following should you expect to occur as a
result?

A Decreased fractional
excretion of sodium

B Increased free water


clearance

C Decreased excretion of
creatinine

D Decreased net acid


secretion

32.53% People got this right

Explanation:

Correct Answer (C)

Decreased excretion of creatinine

Creatinine is freely filtered and slightly secreted. The excretion of creatinine is,
therefore, dependent on filtration, which in turn is dependent on RPF. In CRF,
decrease in GFR results in decrease in creatinine excretion and increase in
plasma creatinine concentration.

To maintain sodium balance, less sodium is reabsorbed. So, the fractional


excretion of sodium (fraction of filtered sodium that is excreted) increases.

Decrease in GFR results in a decrease in free water clearance.

https://emedicoz.com/testresult/683510s14693192 105/176
6/9/24, 12:45 PM Neet PG Preparation, Neet PG Coaching, FMGE, USMLE

In CRF, the remaining nephrons excrete larger than normal amount of H+. But
despite the overall increase in the acid secretion, H+ accumulates in the plasma
leading to metabolic acidosis.

Question: 113
A 60-year-old woman presented with acute onset of right eye pain.
Ophthalmic and neurologic examinations were normal except for a loud
right carotid bruit. The bruit is most likely caused by which of the
following?

A High velocity of blood in


the carotid artery

B Increase in blood viscosity

C Increase in hematocrit

D Increase in diameter of the


carotid artery

45.25% People got this right

Explanation:

Correct Answer (A) High velocity of blood in the carotid artery Bruits are heard
over areas of turbulent flow. The most important factor causing increase in
Reynold’s number and therefore, turbulent flow is velocity. High velocity in a
stenotic area of the carotid artery causes increase in Reynold’s number and
turbulent flow causing the bruit. Increase in hematocrit or blood viscosity would
decrease the Reynold’s number and decrease the tendency for turbulent flow.

Question: 114
Which of the following events normally occur during the PR interval?

https://emedicoz.com/testresult/683510s14693192 106/176
6/9/24, 12:45 PM Neet PG Preparation, Neet PG Coaching, FMGE, USMLE

A Ventricles are contracting

B Cardiac action potential


passes through the AV node

C Mitral and aortic valve are


closed

D Second heart sound is


heard

49.69% People got this right

Explanation:

Correct Answer (B)

Cardiac action potential passes through the AV node

PR interval is from the onset of P to the onset of Q. The physiologic events which
occur during the PR interval are: -

Atrial depolarization (which id responsible for the P wave)


AV nodal depolarization
Depolarization of the bundle of His, bundle branches and Purkinje fibres
The mitral and aortic valves are closed in the isovolumetric contraction phase
which starts after the peak of R. the second heart sound is heard at the end of
ventricular systole which coincides with end of T.

Question: 115
A 60-year-old male with right lower lobar pneumonia presents with
respiratory distress. The patient is intubated and placed on a mechanical
ventilator in the ICU. With the patient positioned on the left side, which
of the following variables will be lower in the left lung as compared to the
right lung?

https://emedicoz.com/testresult/683510s14693192 107/176
6/9/24, 12:45 PM Neet PG Preparation, Neet PG Coaching, FMGE, USMLE

A Alveolar ventilation

B Blood flow

C Lung compliance

D V/Q ratio

20.85% People got this right

Explanation:

Correct Answer (D)

V/Q ratio

The dependent lung will have a higher ventilation and higher blood flow but
lower V/Q ratio. This is because the effect of gravity is greater for blood
(because blood is denser than air), the increase in perfusion exceeds the increase
in ventilation, and the V/Q ratio decreases.

The greater hydrostatic pressure in the dependent lung causes an increased (less
negative) intrapleural pressure, which decreases the lung volume, and places the
pressure-volume curve of the lung on a steeper slope (increased lung
compliance). In patients with unilateral pneumonia, simply positioning the
unaffected lung downward may result in improved ventilation- perfusion
matching and increase in Pao2 of 10-15 mm Hg.

Question: 116
A 30-year-old man, with h/o collapsing suddenly without any other
physical distress, is found to have elevated serum potassium. He is
diagnosed with periodic hyperkalemic paralysis. Which of the following is
the most likely cause for muscle weakness because of increased ECF
potassium level?

https://emedicoz.com/testresult/683510s14693192 108/176
6/9/24, 12:45 PM Neet PG Preparation, Neet PG Coaching, FMGE, USMLE

A Hyperpolarization of
muscle cells

B Inactivation of sodium
channels in muscle cells

C Increased release of
neurotransmitters from α-motor
neurons

D Increased duration of
action potential in muscle cells

11.33% People got this right

Explanation:

Correct Answer (B)

Inactivation of sodium channels in muscle cells

In hyperkalemia, muscle cells are in a persistent depolarization state which


causes inactivation of the sodium channels decreasing their excitability. This
leads to muscle weakness or even paralysis.

Although the exact mechanism of periodic hyperkalemic paralysis is not known,


it may be due to a mutation in the gene coding for the sodium inactivation gate.

Question: 117
Which of the following would be observed in a person who is resting
after an overnight fast?

A Liver glycogen stores are


completely depleted

https://emedicoz.com/testresult/683510s14693192 109/176
6/9/24, 12:45 PM Neet PG Preparation, Neet PG Coaching, FMGE, USMLE

B Liver gluconeogenesis is
not an important process

C Muscle glycogen stores are


used to maintain blood glucose

D Fatty acids are released


from adipose triacylglycerol
stores

20.95% People got this right

Explanation:

Correct Answer (D)

Fatty acids are released from adipose triacylglycerol stores .

Ref– Read the text below

Sol:

Fatty acids are released from adipose tissue and oxidized by other cells. Liver
glycogen is not depleted until about 30 hours of fasting.
After an overnight fast, both glycogenolysis and gluconeogenesis by the liver
help maintain blood glucose.
Muscle glycogen stores are not used to maintain blood glucose.
The liver produces ketone bodies but does not oxidize them.

Question: 118
Which clinical laboratory observation below is suggestive of Hartnup
disease (neutral amino acid transport deficiency)?

A Burnt-sugar smell in urine

B High plasma phenylalanine


levels

https://emedicoz.com/testresult/683510s14693192 110/176
6/9/24, 12:45 PM Neet PG Preparation, Neet PG Coaching, FMGE, USMLE

C Extremely high levels of


citrulline in urine

D High fecal levels of


tryptophan and indole derivatives

27.74% People got this right

Explanation:

Correct Answer (D)

High fecal levels of tryptophan and indole derivatives

Ref– Read the text below

Sol:

In Hartnup disease, a defect in the transport process for neutral amino acids is
most pronounced in intestinal and renal transport.
Neutral aminoaciduria is observed as well as increased fecal excretion of indole
derivations due to bacterial conversion of unabsorbed dietary tryptophan.
Pellagra-like symptoms can be seen due to the lack of tryptophan for niacin
biosynthesis.

Question: 119
Denaturation of DNA is PCR is carried out by heating to a temperature of:

A 40°C

B 60°C

C 76°C

D 94°C

https://emedicoz.com/testresult/683510s14693192 111/176
6/9/24, 12:45 PM Neet PG Preparation, Neet PG Coaching, FMGE, USMLE

40.76% People got this right

Explanation:

Correct Answer (D)

94°C

Ref– Read the text below

Sol:

POLYMERASE CHAIN PEACTION

It is a test tube method for amplifying a selected DNA sequence


The method can be used to amplify DNA sequences from any source:bacterial,
viral, plant or animal
PCR utilizes DNA polymerase to repetitively amplify targeted portions of DNA
Requirements
DNA to be amplified
Primer RNA
Deoxynucleotides
DNA polymerase ( Taq polymerase)
PCR: STEPS

1. Primer construction:

It is not necessary to know the nucleotide sequence of the target DNA


But it is necessary to know the nucleotide sequence of short segments on each
side of target DNA-flanking sequences
2. Denature the DNA:

The DNA to be amplified is heated to separate the double stranded target DNA
into single strands
This is done by heating at 92-96 °C for 10 minutes
3. Annealing of primers to ssDNA:

The separated strands are cooled and allowed to anneal to the two primers (one
each strand)
Temperature : 45 °C for 4 minutes
4. Chain extension:

DNA polymerase and deoxyribonucleoside triphosphates are added to the


mixture to initiate the synthesis of two new chains complementary to the
original DNA
Taq polymerase (from thermusaquaticus) is used
https://emedicoz.com/testresult/683510s14693192 112/176
6/9/24, 12:45 PM Neet PG Preparation, Neet PG Coaching, FMGE, USMLE

At 72 °C for variable time


Generally cycles are repeated for 20-30 times

Question: 120
Which of the following vitamins can be synthesized by intestinal bacteria?

A Vitamin E

B Vitamin C

C Vitamin A

D Vitamin K

48.57% People got this right

Explanation:

Correct Answer (D)

Vitamin K

Ref:Read the text below

Sol:

Intestinal bacteria synthesize vitamin K. The main dietary source of vitamin K is


green leafy vegetables.
Vitamin E, vitamin A, vitamin C, and niacin are not synthesized by intestinal
bacteria.
The main dietary source for vitamin E is vegetable oils. The main dietary sources
for vitamin C are fresh fruit and vegetables. The main dietary sources of vitamin
A are liver, whole milk, fish oils, and eggs.

Question: 121

https://emedicoz.com/testresult/683510s14693192 113/176
6/9/24, 12:45 PM Neet PG Preparation, Neet PG Coaching, FMGE, USMLE

A 40 yr old woman presents with progressive palmoplantar pigmentation.


X-ray spine shows calcification of intervertebral discs. On adding
Benedict’s reagent to urine, it gives greenish brown precipitate and blue
black supernatant fluid. What is the diagnosis?

A Phenylketonuria

B Alkaptonuria

C Tyrosinemia type 2

D Argininosuccinic aciduria

40.93% People got this right

Explanation:

Correct Answer (B)

Alkaptonuria

Ref: Read the text below

Sol:

Alkaptonuria

Alkaptonuria is a rare metabolic disease involving a deficiency in homogentisic


acid oxidase, resulting in the accumulation of homogentisic acid. [Note: This
reaction occurs in the degradative pathway of tyrosine, ]
The illness has three characteristic symptoms: homogentisic aciduria (the
patient's urine contains elevated levels of homogentisic acid, which is oxidized to
a dark pigment on standing,, large joint arthritis, and black ochronotic
pigmentation of cartilage and collagenous tissue .
Patients with alkaptonuria are usually asymptomatic until about age 40.
Dark staining of the diapers sometimes can indicate the disease in infants, but
usually no symptoms are present until later in life.
Diets low in protein—especially in phenylalanine and tyrosine—help reduce the
levels of homogentisic acid, and decrease the amount of pigment deposited in
body tissues.

https://emedicoz.com/testresult/683510s14693192 114/176
6/9/24, 12:45 PM Neet PG Preparation, Neet PG Coaching, FMGE, USMLE

Although alkaptonuria is not life-threatening, the associated arthritis may be


severely crippling.

Question: 122
Which of the following is TNF α blocker used in Rheumatoid arthritis

A Rituximab

B Toclizumab

C Anakinra

D Eternacept

41.39% People got this right

Explanation:

Correct Answer (D)

Tumor necrosis factor (TNF)-α antagonists, such as infliximab (IFX), etanercept


(ETN), adalimumab (ADA), golimumab (GOLI) and certolizumab pegol (CZP) are
used in Rheumatoid arthritis.

Anakinra is interleukin 1 antagonist

Option d is correct here

Question: 123
All are true of Benzodiazpenes except:

A Diazepam is used in the


treatment of status epilepticus

https://emedicoz.com/testresult/683510s14693192 115/176
6/9/24, 12:45 PM Neet PG Preparation, Neet PG Coaching, FMGE, USMLE

B Diazepam is used in long


term treatment of psychic
disorders

C Clonazepam is used in the


treatment of petit mal epilepsy

D None of the above

28.38% People got this right

Explanation:

Correct Answer (B) Benzodiazepines show a side effect of tolerance on chronic


use so it is not advised for long term treatment of psychic disorder. Lorazepam is
DOC for status epilepticus clonazepam can be used in petit mal epilepsy
management . option B is correct here . e.g. in mania though treatment is
started with lorazepam but is never continued lifelong.

Question: 124
Morphine is

A Sedative

B Alkaloid

C Diuretic

D Antihypertensive

24.54% People got this right

Explanation:

Correct Answer (B)

https://emedicoz.com/testresult/683510s14693192 116/176
6/9/24, 12:45 PM Neet PG Preparation, Neet PG Coaching, FMGE, USMLE

Morphine is the principal alkaloid in opium and is widely used till today.

Question: 125
Drug which does not inhibit action by competitive inhibition

A Organophosphate

B Sulfonamides

C Acetazolamide

D Allopurinol

21.25% People got this right

Explanation:

Correct Answer (C)

Acetazolamide

https://emedicoz.com/testresult/683510s14693192 117/176
6/9/24, 12:45 PM Neet PG Preparation, Neet PG Coaching, FMGE, USMLE

Question: 126
If patient is given noradrenaline what will happen

A Causes inc in BP due to its


action on beta receptors only

B Cause decrease in BP due


to its action on alpha receptors
only

C Causes increase in cardiac


output and reflex bradycardia

D None of the above

31.6% People got this right

Explanation:

Correct Answer (C) Causes increase in cardiac output and reflex bradycardia
Goodman and Gilman: In response to intravenous infusion of NE in humans,
systolic and diastolic pressures, and usually pulse pressure, are increased.
Cardiac output is unchanged or decreased, and total peripheral resistance is
raised. Compensatory vagal reflex activity slows the heart producing bradycardia
, overcoming a direct cardioaccelerator action, and stroke volume is increased.
The peripheral vascular resistance increases in most vascular beds, and renal
blood flow is reduced. NE constricts mesenteric vessels and reduces splanchnic

https://emedicoz.com/testresult/683510s14693192 118/176
6/9/24, 12:45 PM Neet PG Preparation, Neet PG Coaching, FMGE, USMLE

and hepatic blood flow. Coronary flow usually is increased, probably owing both
to indirectly induced coronary dilation, as with EPI, and to elevated blood
pressure. Although generally a poor β2 receptor agonist, NE may increase
coronary blood flow directly by stimulating β2 receptors on coronary ves- sels.
Patients with Prinzmetal variant angina may be supersensitive to the α
adrenergic vasoconstrictor effects of NE.

Question: 127
A patient on 300 mg of aspirin will show all the following except

A Irreversible inhibition of
cyclooxygenase pathway

B Prolonged bleeding time

C Inhibition of thromboxane
TXA2

D Inhibition of prostaglandin
PGI2

26.94% People got this right

Explanation:

Correct Answer (D)

Inhibition of prostaglandin PGI2

Aspirin, even in small doses, irreversibly inhibits cyclooxygenase enzyme


reducing TXA2 synthesis by platelets. Thus, it interferes with platelet aggregation
and bleeding time is prolonged to nearly twice the normal value. This effect lasts
for about a week (turnover time of platelets).

NSAIDs inhibit synthesis of both proaggregatory (TXA2) and anti- aggregatory


(PGI2) prostanoids, but effect on platelet TXA2 (COX-1 generated) predominates

https://emedicoz.com/testresult/683510s14693192 119/176
6/9/24, 12:45 PM Neet PG Preparation, Neet PG Coaching, FMGE, USMLE

→therapeutic doses of most NSAIDs inhibit platelet aggregation: bleeding time


is prolonged.

Question: 128
LA act by inhibiting

A Motor fibre only

B Motor and sensory fibres

C Only sensory fibres

D All

13.63% People got this right

Explanation:

Correct Answer (D) All Local anaesthetics (LAs) are drugs which upon topical
application or local injection cause rever- sible loss of sensory perception,
especially of pain, in a restricted area of the body. They block generation and
conduction of nerve impulse at any part of the neurone with which they come in
contact, without causing any structural damage. Thus, not only sensory but also
motor impulses are interrupted when a LA is applied to a mixed nerve, resulting
in muscular paralysis and loss of autonomic control as well.

Question: 129
Which of the following is not a broad spectrum antibiotic?

A Tetracycline

B Chloramphenicol

https://emedicoz.com/testresult/683510s14693192 120/176
6/9/24, 12:45 PM Neet PG Preparation, Neet PG Coaching, FMGE, USMLE

C Doxycycline

D Cephalosporins

13.55% People got this right

Explanation:

Correct Answer (C)

Cephalosporins

Question: 130
Minimum amount of tetracycline required for tooth discolor

A 1mg / kg body weight

B 10 mg / kg body weight.

C 21mg / kg body weight

D 50 mg / kg body weight

23.76% People got this right

https://emedicoz.com/testresult/683510s14693192 121/176
6/9/24, 12:45 PM Neet PG Preparation, Neet PG Coaching, FMGE, USMLE

Explanation:

Correct Answer (B)

Question: 131
Which of the following benzodiazepine is safe in liver disease

A Diazepam

B Lorazepam

C Clobazam

D Midazolam

32.09% People got this right

Explanation:

Correct Answer (B) Lorazepam Triazolam, oxazepam, temazepam, estazolam and


lorazepam are safe in liver failure

Question: 132
" Holistic concept " of health deals with :

A Absence of disease

B Man and environment

C Sound mind in a sound


body in a sound environment

https://emedicoz.com/testresult/683510s14693192 122/176
6/9/24, 12:45 PM Neet PG Preparation, Neet PG Coaching, FMGE, USMLE

D None of these

54.6% People got this right

Explanation:

Correct Answer (C)

" Holistic concept " of health : Sound mind in a sound body in a sound
environment

Absence of disease : Biological concept

Man and environment : Ecological concept

Question: 133
Primary prevention includes :

1- Health education

2-Health promotion

3- Specific protection

4- Breast self examination

5- Reconstructive surgery

A Only 1

B 1, 2

C 1,2,3

D 1, 2, 3, 5

45.96% People got this right

https://emedicoz.com/testresult/683510s14693192 123/176
6/9/24, 12:45 PM Neet PG Preparation, Neet PG Coaching, FMGE, USMLE

Explanation:

Correct Answer (C)

PRIMARY PREVENTION-

Intervention in pre – pathogenesis phase

Risk factors are present but disease has not yet taken place
Modes of intervention
A) HEALTH PROMOTION
Health education, Environmental modification, lifestyle & behavioral changes
EX- Insecticides spray, Potable safe water supply, Life style modification, Personal
hygiene and Environmental sanitation

B) SPECIFIC PROTECTION – Specific intervention. E.g. Immunization,


chemoprophylaxis, use of specific nutrient, Usage of condoms, Iodisation of salt,
deflouridation

Question: 134
An example for cyclo-propogative transmission is :

A Plague

B Malaria

C Filariasis

D Rabies

37.09% People got this right

Explanation:

Correct Answer (B) Cyclo-propogative transmission is : Both multiplication and


development of agent Ex: Plasmodium within anopheles

https://emedicoz.com/testresult/683510s14693192 124/176
6/9/24, 12:45 PM Neet PG Preparation, Neet PG Coaching, FMGE, USMLE

Question: 135
Calculate vaccine efficacy from the table shown below :

A 58%

B 78%

C 88%

D 98%

24.35% People got this right

Explanation:

Correct Answer (B)

Attack rate in un-vaccinated (ARU) : 9/10 = 0.9

Attack rate in vaccinated (ARV): 2/10 = 0.2

Vaccine efficacy = (ARU-ARV)/ARU

=(0.9-0.2)/0.9

= 0.7/0.9

= 78%

https://emedicoz.com/testresult/683510s14693192 125/176
6/9/24, 12:45 PM Neet PG Preparation, Neet PG Coaching, FMGE, USMLE

Question: 136
Pearson coefficient between salt intake and blood pressure is 0.8. Which
of the following is TRUE

A It indicates obesity is a risk


factor for hypertension

B The relationship is
represented by histogram

C It indicates positive
correlation

D All of these

33.97% People got this right

Explanation:

Correct Answer (C)

Pearson coefficient:

Pearson coefficient is correlation coefficient


Pearson coefficient between salt intake and blood pressure is 0.8 :
So, It indicates positive correlation
The corelation is represented by scatter diagram
Correlation does not measure risk . So it will not indicate obesity is a risk factor
for hypertension

Question: 137
The investigator wants to determine whether exposure to chemicals used
in tire manufacturing was associated with an increased risk of death. He
finds a tire manufacturing factory that had been in operation for several
decades. He used employee health records to identify those who had had
jobs which involved exposure to the chemicals in question (workers who

https://emedicoz.com/testresult/683510s14693192 126/176
6/9/24, 12:45 PM Neet PG Preparation, Neet PG Coaching, FMGE, USMLE

actually manufactured tires) and non-exposed coworkers (clerical workers


or sales employees that didn't involve exposure to the chemicals). He
then ascertained what had happened to all the subjects and compare the
incidence of death in the exposed and non-exposed workers. What is this
study :

A Prospective cohort study

B Retrospective cohort study

C Case control study

D Cross sectional study

46.59% People got this right

Explanation:

Correct Answer (B)

Question: 138
A pharmaceutical company develops a new anti-hypertensive drug.
Samples of 24 hypertensive patients, randomly selected from a large
population of hypertensive people, are randomly divided into 2 groups of
12. One group is given the new drug over a period of 1 month; the other
group is given a placebo according to the same schedule. Neither the
https://emedicoz.com/testresult/683510s14693192 127/176
6/9/24, 12:45 PM Neet PG Preparation, Neet PG Coaching, FMGE, USMLE

patients nor the treating physician are aware of which patients are in
which group . At the end of the month, measurements are made of the
patient’s blood pressures. This study-

A Is a randomized controlled
clinical trial

B Uses a crossover design

C Is a single blind experiment

D Is a retrospective study

56.12% People got this right

Explanation:

Correct Answer (A) Is a double blinded concurrent parallel randomized


controlled clinical trial

Question: 139
Mobile health teams to screen for deficiencies in children is appointed
under :

A JSSK

B RKSK

C RBSK

D NSSK

51.01% People got this right

https://emedicoz.com/testresult/683510s14693192 128/176
6/9/24, 12:45 PM Neet PG Preparation, Neet PG Coaching, FMGE, USMLE

Explanation:

Correct Answer (C)

Question: 140
"SPARSH" campaign is to create awareness for :

A Leprosy

B TB

C AIDS

D STDs

54.24% People got this right

Explanation:

Correct Answer (A) "SPARSH" campaign is to create awareness for : Leprosy

Question: 141
All are morbidity indicators except :

https://emedicoz.com/testresult/683510s14693192 129/176
6/9/24, 12:45 PM Neet PG Preparation, Neet PG Coaching, FMGE, USMLE

A Incidence

B Duration of hospital stay

C Notification rates

D Years of potential life lost

21.66% People got this right

Explanation:

Correct Answer (D) Morbidity indicators Incidence , Prevalence , Duration of


hospital stay , Notification rates Years of potential life lost : To express premature
mortality . Used in DALYs

Question: 142
Out of cases 80% are exposed and out of controls 80% are non exposed.
Calculate odds ratio:

A 12

B 14

C 16

D 18

47.45% People got this right

Explanation:

Correct Answer (C)

https://emedicoz.com/testresult/683510s14693192 130/176
6/9/24, 12:45 PM Neet PG Preparation, Neet PG Coaching, FMGE, USMLE

Question: 143
A new treatment can reduce mortality in cancer patients from 17% to 12%
. Calculate NNT:

A 5

B 10

C 15

D 20

25.25% People got this right

Explanation:

Correct Answer (D)

A new treatment can reduce mortality in cancer patients from 17% to 12% .

So , Absolute risk reduction(ARR) is : 17 – 12 = 5%

NNT = 100 / ARR


= 100 / 5
= 20

https://emedicoz.com/testresult/683510s14693192 131/176
6/9/24, 12:45 PM Neet PG Preparation, Neet PG Coaching, FMGE, USMLE

Question: 144
Which of the following is true for the table shown below :

A Sensitivity is same as
Specificity

B Sensitivity is more than


Specificity

C Sensitivity is less than


Specificity

D None of these

43.88% People got this right

Explanation:

Correct Answer (C) Explanation : SENSITIVITY = 37 / 59 = 62% SPECIFICITY = 130


/ 144 = 91% Sensitivity is less than Specificity

Question: 145
The error of not rejecting a false null hypothesis is :

https://emedicoz.com/testresult/683510s14693192 132/176
6/9/24, 12:45 PM Neet PG Preparation, Neet PG Coaching, FMGE, USMLE

A Type I error

B Type II error

C Both of these

D None of these

33.14% People got this right

Explanation:

Correct Answer (B)

Type II error :

The error of not rejecting a false null hypothesis


In a study no difference is observed while actually there is true difference
False negative error

Question: 146
Which of the following adverse effect of vaccines is wrongly matched :

A 17D : Vaccine associated


viscerotropic disease

B Measles : Toxic shock


syndrome

C Influenza (inactivated) :
Guillain-Barre syndrome

D IPV : VAPP

https://emedicoz.com/testresult/683510s14693192 133/176
6/9/24, 12:45 PM Neet PG Preparation, Neet PG Coaching, FMGE, USMLE

31.18% People got this right

Explanation:

Correct Answer (D)

17D : Vaccine associated viscerotropic disease


Measles : Toxic shock syndrome
Influenza (inactivated) : Guillain-Barre syndrome
OPV : VAPP

Question: 147
Which of the following non depolarizing muscle relaxant is short acting

A Atracurium

B Rocuronium

C Succinylcholine

D Gantacurium

21.17% People got this right

Explanation:

Correct Answer (D)

Question: 148
Rising end tidal CO2 may be seen in all the following conditions except

https://emedicoz.com/testresult/683510s14693192 134/176
6/9/24, 12:45 PM Neet PG Preparation, Neet PG Coaching, FMGE, USMLE

A Pulmonary thrombus

B Sepsis

C Depleted CO2 absorber

D Malignant hyperthermia

29.18% People got this right

Explanation:

Correct Answer (A)

Question: 149
The following ventilator mode doesnot allow for spontaneous respiration

A SIMV

B HFJV

C PSV

D CMV

23.83% People got this right

Explanation:

Correct Answer (D)

https://emedicoz.com/testresult/683510s14693192 135/176
6/9/24, 12:45 PM Neet PG Preparation, Neet PG Coaching, FMGE, USMLE

Question: 150
Evoked potentials are most affected by

A Nitrous oxide

B Volatile anaesthetics

C Opoids

D Non depolarising muscle


relaxants

12.16% People got this right

Explanation:

Correct Answer (B)

Question: 151
A patient presents with mucoid discharge from urethra with dysuria since
1 day. He had an unprotected sexual exposure 14 days back. The
treatment of choice would be:

A Cefixime

B Penicillin

C Azithromycin

D Cotrimoxazole

32.16% People got this right

https://emedicoz.com/testresult/683510s14693192 136/176
6/9/24, 12:45 PM Neet PG Preparation, Neet PG Coaching, FMGE, USMLE

Explanation:

Correct Answer (C) The diagnosis is non gonococcal urethritis for which
azithromycin 1 gm stat dose would suffice.

Question: 152
A patient presented with lesions as shown. What is the microscopic finding
observed in this patient?

A Prominent necrotic cell in


granular layer

B Suprabasal split

C Hypergranulosis

D Acantholysis

25.08% People got this right

Explanation:

Correct Answer (C) The lesions are of lichen planus. Note the flexural
involvement. Hypergranulosis will be seen.

https://emedicoz.com/testresult/683510s14693192 137/176
6/9/24, 12:45 PM Neet PG Preparation, Neet PG Coaching, FMGE, USMLE

Question: 153
A 60 year old patient presents with a lesion on the skin as shown. What is
the likely diagnosis?

A Basal cell carcinoma

B Squamous cell carcinoma

C Actinic Keratosis

D Warts

53.94% People got this right

Explanation:

Correct Answer (A) Note the site (face) above the line connecting the angle of
the mouth and the outer canthus. Intermittent intense sun exposure, as
identified by prior sunburns; radiation therapy; a positive family history of BCC;
immunosuppression; a fair complexion, especially red hair; easy sunburning (skin
types I or II); are risk factors for the development of BCC. The classic or nodular
BCC comprises 50–80% of all BCCs. Nodular BCC is composed of one or a few
small, waxy, semitranslucent nodules, forming around a central depression that
may or may not be ulcerated, crusted, and bleeding. The edge of larger lesions
has a characteristic rolled border. Telangiectases course through the lesion.
Bleeding on slight injury is a common sign.

https://emedicoz.com/testresult/683510s14693192 138/176
6/9/24, 12:45 PM Neet PG Preparation, Neet PG Coaching, FMGE, USMLE

Question: 154
Which of the following is not a type of lichen planus?

A Lichen scrofulosorum

B Lichen planopilaris

C Lichen hypertrophica

D Lichen pigmentosa

36.9% People got this right

Explanation:

Correct Answer (A) Lichen scrofulosorum is a type of tuberculid

Question: 155
What is incorrect about androgenetic alopecia?

A Males and females present


with hair line recession

B Finasteride is used to treat


androgenetic alopecia in males

C Minoxidil is a useful topical


agent

D Histology shows
miniaturization of hairs

44.3% People got this right


https://emedicoz.com/testresult/683510s14693192 139/176
6/9/24, 12:45 PM Neet PG Preparation, Neet PG Coaching, FMGE, USMLE

Explanation:

Correct Answer (A) Females do not have hair line recession

Question: 156
Which of the following is not true regarding anatomy of pelvis?

A Ovarian arteries arise from


abdominal aorta

B Ovarian veins drain into


inferior vena cava

C Uterine arteries arise from


anterior division of internal iliac

D Cornual area of the uterus


drains into superficial inguinal
lymph nodes

21.47% People got this right

Explanation:

Correct Answer (B)

Ovarian veins drain into inferior vena cava

Left ovarian vein drains into left renal vein and right ovarian vein drains into IVC.
Other statements are correct
(ref: William’s obstetrics, 25th edition, chapter: Maternal anatomy)

Question: 157
Which one of the following is not Mullerian in origin?

https://emedicoz.com/testresult/683510s14693192 140/176
6/9/24, 12:45 PM Neet PG Preparation, Neet PG Coaching, FMGE, USMLE

A Epithelium of vagina

B Hydatid of morgagni

C Appendix of testes

D Prostatic utricle

24.62% People got this right

Explanation:

Correct Answer (A)

Epithelium of vagina

Vagina originates from Mullerian as well as Urogenital sinus. Upper part is


formed by Mullerian and the lower part is formed by Urogenital sinus. However,
the epithelium of whole vagina originates from Urogenital sinus.

(ref: William’s obstetrics, 25th edition, chapter: Maternal anatomy)

Question: 158
Identify the condition. This patient came in 4th month of pregnancy with
history of bleeding per vaginum.

https://emedicoz.com/testresult/683510s14693192 141/176
6/9/24, 12:45 PM Neet PG Preparation, Neet PG Coaching, FMGE, USMLE

A Abruptio placenta

B Cervical incompetence

C Partial molar pregnancy

D Low lying placenta

49.65% People got this right

Explanation:

Correct Answer (C)

Question: 159
A 25 years old primigravida with tubal ectopic pregnancy of 6 weeks was
subjected to single dose therapy (medical management). Her b-Hcg value
is 2500 IU/L. After three days of methotrexate injection, her b-Hcg is 3250
IU/L. What is the next best line of management?

A Give inj. Methotrexate


again

B Repeat the b-Hcg test after


3 days

https://emedicoz.com/testresult/683510s14693192 142/176
6/9/24, 12:45 PM Neet PG Preparation, Neet PG Coaching, FMGE, USMLE

C Laparoscopic
salpingostomy

D PGF2a injection

28.86% People got this right

Explanation:

Correct Answer (B)

Repeat the b-Hcg test after 3 days

The patient is a case of unruptured ectopic for which single dose medical
management is given. The baseline HCG is 2500 IU/L, the day4 HCG was raised
but it will not decide the mode of further treatment. You will have to repeat HCG
after 3 more days and this HCG of day & will decide the further management.

(ref: William’s obstetrics, 25th edition, chapter: ectopic pregnancy)

Question: 160
While conducting vaginal delivery of breech presentation, it was found
that he back of the baby was towards the sacrum of the mother during
delivery of after-coming head. Which of the following manoeuvre is used
for this situation?

A Piper’s forceps method

B MSV manoeuvre

C Prague manoeuvre

D Bracht manoeuvre

33.65% People got this right

https://emedicoz.com/testresult/683510s14693192 143/176
6/9/24, 12:45 PM Neet PG Preparation, Neet PG Coaching, FMGE, USMLE

Explanation:

Correct Answer (C)

Prague manoeuvre

Piper’s forceps are used for the entrapped after-coming head with dorso-
posterior position. MSV method is jaw flexion and shoulder traction. Bracht
method is to put the fetus on pubic symphysis and to give suprapubic thrust.
Prague is the only method which is done in a dorso posterior position. Baby is
held by legs and then it is moved in the long arc with suprapubic procedure to
maintain flexion.

(ref: William’s obstetrics, 25th edition, chapter: breech delivery)

Question: 161
While doing caesarean section, it was found that there was atonic PPH
which was not responding to uterotonics. One of the following is not a
correct option in the management of this patient?

A B-lynch suture

B Anterior division of internal


iliac artery ligation

C Uterine balloon
tamponade

D Subtotal hysterectomy

20.2% People got this right

Explanation:

Correct Answer (C)

Uterine balloon tamponade

https://emedicoz.com/testresult/683510s14693192 144/176
6/9/24, 12:45 PM Neet PG Preparation, Neet PG Coaching, FMGE, USMLE

Uterine balloon tamponade is a method in which condom or any other balloon


is put inside uterus through vagina and then it is distended to give a
compression effect on the bleeding decidual sinuses. This is not possible when
caesarean section is done and if medical management fails.

(ref: William’s obstetrics, 25th edition, chapter: obstetrical hemorrhage)

Question: 162
A 25-year-old primigravida with 32 weeks of gestation has got bp of
160/90 mm of hg. Her urine albumin is 1+. She has no other symptoms
and all her investigations are normal. Her 1st trimester bp was normal.
What is your diagnosis?

A Take 2nd reading 4 hours


later

B Gestational hypertension

C Pre-eclampsia

D Chronic hypertension

16.41% People got this right

Explanation:

Correct Answer (B)

Gestational hypertension

This is a case of gestational hypertension because the proteinuria for pre


eclampsia should be 2+ or above. Second reading is not necessary if her systolic
BP is 160 or a diastolic BP is 110mm Hg.

(ref: William’s obstetrics, 25th edition, chapter: Hypertension in pregnacy)

https://emedicoz.com/testresult/683510s14693192 145/176
6/9/24, 12:45 PM Neet PG Preparation, Neet PG Coaching, FMGE, USMLE

Question: 163
Which one of the following is not seen in twin anemia-polycythemia
sequence?

A Oligo-polyhydramnios

B AV malformation in
monochorionic twins

C Fetoplacental insufficiency

D Hydrops fetalis

8.96% People got this right

Explanation:

Correct Answer (A)

Oligo-polyhydramnios

TAPS (TWIN ANEMIA POLYCYTHEMIA) sequence is same as TTTs. It is also due to


abnormal AV malformation in monochorionic twins. The only difference betweer
these two is that TAPS does not have oligo-poly complex.

(ref: William’s obstetrics, 25th edition, chapter: Multifetal gestation )

Question: 164
During delivery of Rh-negative pregnancy with ICT negative status, there
was abruptio placenta and o positive baby was delivered by caesarean
section. Her Kleihauer-Betke test revealed 20 ml fetal RBC in maternal
circulation. How much Anti-D should be given?

A 200 mcg

https://emedicoz.com/testresult/683510s14693192 146/176
6/9/24, 12:45 PM Neet PG Preparation, Neet PG Coaching, FMGE, USMLE

B 300 mcg

C 400 mcg

D No need of anti-D

16.92% People got this right

Explanation:

Correct Answer (C)

400 mcg

In an ICT negative patient, we need to prevent isoimmunisation by giving Anti-D


injection ideally within 72 hours of delivery of Rh positive baby. In case of severe
fetomaternal mixing, a quantitative test, called as KB test is done to determine
the dose of Anti D required. For 1ml of fetal blood 10 mcg of Anti-D is required
to neutralise it. 1ml of fetal blood means 0.5ml of fetal RBC. For 20 ml of fetal
RBC i.e 40 ml of fetal blood 400mcg of Anti-D is needed.

(ref: Dutta obstetrics, chapter: special topics: Rh negative pregnancy)

Question: 165
Which one of the following is not correct in down’s syndrome screening?

A Free b-Hcg is raised

B Inhibin-A is raised

C PAPP-A is raised

D AFP is reduced

https://emedicoz.com/testresult/683510s14693192 147/176
6/9/24, 12:45 PM Neet PG Preparation, Neet PG Coaching, FMGE, USMLE

35.29% People got this right

Explanation:

Correct Answer (C)

PAPP-A is raised

Trisomy sceenings are of two types: 1st trimester screening which is also called
as combined screening of T1. It consists of NT scan (critical cut off is 3mm) and
dual marker test which is made up of Free B-HCG (raised in Down’s syndrome)
and PAPPA (decreased in Down’s syndrome). In T2 screening, there is Quadruple
marker test which consists of Free B-HCG (raised in T21), AFP (decreased in T21),
unconjugated estriol (decreased in T21) and inhibin-A (raised in T21).

(ref: William’s obstetrics, 25th edition, chapter: prenatal diagnosis)

Question: 166
A 34-year-old primigravida has IUGR with abdominal circumference of
the baby of 5th percentile. She is having 35 weeks of gestation. Her USG
doppler was done which showed umbilical artery PULSATILITY INDEX of
2.1. What should be our next plan of action?

A Continue pregnancy till 38


weeks

B Give steroids and continue


till 37 weeks

C Give steroids and do


induction of labour immediately

D Give steroids and do LSCS


immediately.

11.7% People got this right

https://emedicoz.com/testresult/683510s14693192 148/176
6/9/24, 12:45 PM Neet PG Preparation, Neet PG Coaching, FMGE, USMLE

Explanation:

Correct Answer (D)

Give steroids and do LSCS immediately.

Doppler usg is an important investigation which provides information about


fetoplacental insufficiency. Umbilical artery doppler is used for the same.
Pulsatility index is a marker of resistance. It means (S-D)/Average of S and D
In a case of absent diastolic flow, the value of D is 0 so the PI would be 2.0. if PI
is more than 2.0, it means there is reversal of diastolic flow. In such cases, we do
caesarean section at the end of 32 weeks. In this case, patient has 35 weeks of
gestation so we must give steroids and do immediate caesarean section.

(ref: William’s obstetrics, 25th edition, chapter: FETAL GROWTH- DISORDERS)

Question: 167
A 38-week gestational age baby was born with clitoromegaly and
labioscrotal fusion. Which one of the following is not a possible cause?

A Congenital adrenal
hyperplasia

B Partial androgen
insensitivity syndrome

C Fraser syndrome

https://emedicoz.com/testresult/683510s14693192 149/176
6/9/24, 12:45 PM Neet PG Preparation, Neet PG Coaching, FMGE, USMLE

D Swyer syndrome

25.74% People got this right

Explanation:

Correct Answer (D)

Swyer syndrome

The most common cause of ambiguous genitalia at birth is congenital adrenal


hyperplasia which is cortisol pathway defect. Partial AIS is an X linked recessive
disease with defect in the androgenic receptors. There are some receptors
present so the baby is born with ambiguous genitalia. Fraser syndrome is a
genetic disorder with cryptophthalmia and ambiguous genitalia. Swyer
syndrome is a pure gonadal dysgenesis so genotypical male baby is having SRY
gene defect and hence the phenotype of this baby is female. Ambiguous
genitalia is not seen in this case.

(ref: William’s gynecology, 4th edition, disorders of sexual development)

Question: 168
Which one of the following is not a hypogonadotropic hypogonadism:

A Kallmann’s syndrome

B Sheehan’s syndrome

C Klienfelter syndrome

D Prolactinoma

35.8% People got this right

Explanation:

https://emedicoz.com/testresult/683510s14693192 150/176
6/9/24, 12:45 PM Neet PG Preparation, Neet PG Coaching, FMGE, USMLE

Correct Answer (C)

Klienfelter syndrome

Klienfelter syndrome is 47XXY with genotypically male person with tall height,
long limbs but testicular failure. Sr. testosterone is very low and so the
gonadotropins are very high. It is considered as hypergonadotropic
hypogonadism.

(ref: William’s gynecology, 4th edition, disorders of sexual development)

Question: 169
Which one of the following is not indicative of ovulatory cycle:

A Endometrial biopsy
showing subnuclear vacuolation

B Vaginal cytology shows


predominance of intermediate
cells

C Sr. Progesterone >3ng/ml


on day 22 of a menstrual cycle

D Sr. FSH > 40IU/L

41.27% People got this right

Explanation:

Correct Answer (D)

Sr. FSH > 40IU/L

Sr. FSH more than 40IU/L is a marker of menopause with ovarian failure. It is not
possible to have ovulation and menstruation in such cases.

https://emedicoz.com/testresult/683510s14693192 151/176
6/9/24, 12:45 PM Neet PG Preparation, Neet PG Coaching, FMGE, USMLE

(ref: Speroff gynecology, topic: ovarian factor of infertility)

Question: 170
A 33 years old G3P2L2 with 30 weeks of gestation with previous full term
normal deliveries was incidentally diagnosed as a case of short cervix
with cervical length of 1.5 cm. There is no other obstetric high risk and
clinical examination is within normal limits. Which of the following is a
correct treatment plan for this patient?

A 17- oh progesterone
weekly with tocolytics

B 17- oh progesterone with


cervical cerclage

C Cervical cerclage

D 17-oh progesterone

28.4% People got this right

Explanation:

Correct Answer (D) 17-oh progesterone In a case of isolated shortening of cervix


when there is no history of past preterm delivery or painless abortion of second
trimester, there is no need of cervical cerclage. This patient will have high risk of
repeat preterm delivery for which we must give weekly progesterone injections
till term. (ref: William’s obstetrics, 25th edition, chapter: abortions)

Question: 171
A 25-year-old female has invasive mole which is invading the outer half of
myometrium. It has not gone beyond that. Her prognostic score is 3. She
wants to conserve her fertility for future child bearing function. What is
the best plan of action in this case?

https://emedicoz.com/testresult/683510s14693192 152/176
6/9/24, 12:45 PM Neet PG Preparation, Neet PG Coaching, FMGE, USMLE

A Advise her hysterectomy


since invasive mole is a cancer

B Give 4 doses of
methotrexate therapy and advise
her contraception for 1 year after
HCG is undetectable

C Give EMACO regimen and


advise her contraception for 1
year after HCG is undetectable

D Give methotrexate after


doing hysterectomy.

28.28% People got this right

Explanation:

Correct Answer (B) Give 4 doses of methotrexate therapy and advise her
contraception for 1 year after HCG is undetectable For a GTN with low risk (score
less than 7) and whose family is not complete, the treatment is single drug
chemotherapy by methotrexate (4 doses) followed by weekly HCG monitoring till
negative for 3 weeks followed by monthly HCG for 12 months. So the patient is
advised to use COC pills for 12 months. Uterus is not removed if family is not
complete. (ref: William’s obstetrics, 25th edition, chapter: GTD)

Question: 172
Which one of the following is not true regarding emergency
contraception?

A Ulipristal acetate 30 mg
single tab is valid till 5 days of
unprotected intercourse

https://emedicoz.com/testresult/683510s14693192 153/176
6/9/24, 12:45 PM Neet PG Preparation, Neet PG Coaching, FMGE, USMLE

B Copper t prevents
implantation and it acts within
120 hours of unprotected
intercourse

C LNG 150 mcg one tab or


75 mcg 2 tabs 12 hours apart are
used but these are slightly less
effective than ulipristal acetate

D Yuzpe method is high dose


COC pills (2 stat and 2 after 12
hours)

17.58% People got this right

Explanation:

Correct Answer (C)

LNG 150 mcg one tab or 75 mcg 2 tabs 12 hours apart are used but these are
slightly less effective than ulipristal acetate

Levonorgestrel is an emergency contraceptive which is given within 72 hours of


unprotected intercourse. Its dose is 1500mcg (one tablet) or 750 mcg (one stat
and one after 12 hours)

(ref: Dutta Gynecology, chapter: contraception)

Question: 173
If a patient with cu-t has missed her period and she is pregnant. Patient
does not want to undergo MTP. Which of the following is not true
regarding this patient?

A If the thread is visible then


the cu-t must be removed

https://emedicoz.com/testresult/683510s14693192 154/176
6/9/24, 12:45 PM Neet PG Preparation, Neet PG Coaching, FMGE, USMLE

B If the thread is not visible


then artery forceps/IUD hook
must be used to remove cu-t

C Patient has high risk of


getting spontaneous abortion
and preterm labour

D There is no risk of
congenital anomaly to the fetus

38.14% People got this right

Explanation:

Correct Answer (B) If the thread is not visible then artery forceps/IUD hook must
be used to remove cu-t If there is a failure of contraception, then ideally the
patient can undergo MTP but if patient wants to continue pregnancy then Cu-T
should be removed only if the tread is visible on examination since there is high
risk of abortions, preterm labour with Cu-T. it does not increase the risk of
anomalies. If the treat is not visible then it should be left in situ because the use
of any instrument to remove Cu-T can trigger abortion. (ref: William’s obstetrics,
25th edition, contraception and sterilisation)

Question: 174
Which of the following sonographical findings are suggestive of
malignant ovarian etiology?

A Uniloculated cyst

B Papillary projections

C High resistance in doppler


usg

https://emedicoz.com/testresult/683510s14693192 155/176
6/9/24, 12:45 PM Neet PG Preparation, Neet PG Coaching, FMGE, USMLE

D All of the above

28.96% People got this right

Explanation:

Correct Answer (B)

Papillary projections

Papillary projections, solid component, large size, thick irregular septations with
increased vascularity, multilocular cyst, ascites and lymph nodes are markers of
malignant ovarian cysts on USG

(ref: William’s gynecology, 4th edition, ovarian tumours)

Question: 175
A 25 years old unmarried nulliparous patient had acute abdomen. Usg
revealed bilateral dermoid cysts of 8 cm with left ovarian torsion. Which
of the following is best for her?

A Un-torsion and bilateral


cystectomy

B Left oophorectomy and


right cystectomy

C Bilateral oophorectomy
and HRT

D Un-torsion and fixing the


left ovary with round ligament

24.52% People got this right

Explanation:

https://emedicoz.com/testresult/683510s14693192 156/176
6/9/24, 12:45 PM Neet PG Preparation, Neet PG Coaching, FMGE, USMLE

Correct Answer (A) Un-torsion and bilateral cystectomy Dermoids are benign
cystic teratomas which are found in reproductive age. Most common
complication is torsion. In these patients, we must save the ovaries if these
ovaries are viable. In most cases of torsion, the ovarian tissue will not be
necrosed so we must do untorsion and cystectomy as the treatment of choice.
(ref: William’s gynecology, 4th edition, ovarian tumours)

Question: 176
According to MTP act (amendment 2020, passed in 2021 which came into
effect from 24th September 2021, MTP for contraceptive failure is allowed
till?

A 20 weeks and 1 doctor is


needed

B 24 weeks and 2 doctors are


needed

C 24 weeks and 1 doctor is


needed

D 20 weeks and 2 doctors are


needed

24.59% People got this right

Explanation:

Correct Answer (A) 20 weeks and 1 doctor is needed The NEW AMENDMENT
says that MTP can be done till 24 weeks but for contraceptive failure, the limit is
not changed. It is still 20 weeks. Only one doctor is needed now till 20 weeks as
compared to 2 doctors beyond 12 weeks. (ref: THE GAZETTE OF INDIA, MTP ACT
AMENDMENT)

Question: 177

https://emedicoz.com/testresult/683510s14693192 157/176
6/9/24, 12:45 PM Neet PG Preparation, Neet PG Coaching, FMGE, USMLE

Eligibility for tubal ligation includes all except?

A Age >22 years and <49


years

B Ever-married

C At least 2 children with


younger one should be more than
1 year of age

D Consent of spouse is not


needed

25.05% People got this right

Explanation:

Correct Answer (C) At least 2 children with younger one should be more than 1
year of age Tubal ligation is allowed in India only if the patient has one baby of
at least 1 year of age. (guidelines by National family welfare programme)

Question: 178
A 28 years old G3P2L2 with 32 weeks of gestation has come with history
of something coming out per vaginum since last 2-3 weeks. On
examination, there is uterine prolapse of grade 3 with no cystocoele or
rectocoele. There is stress urinary incontinence as well. What should be
the best modality of treatment?

A Left lateral position and


bed rest with catheterization

B Sling surgery with tot

https://emedicoz.com/testresult/683510s14693192 158/176
6/9/24, 12:45 PM Neet PG Preparation, Neet PG Coaching, FMGE, USMLE

C Tot with Kegel’s exercise

D Vaginal pessary

28.74% People got this right

Explanation:

Correct Answer (D) Vaginal pessary In a case of pregnancy with prolapse, surgery
for prolapse is not indicated. The case should be treated with ring pessary with
knob for stress urinary incontinence. (ref: TeLinde’s operative gynecology,
urogynecology)

Question: 179
One of the following is not a part of Amsel criteria?

A pH <4.5

B Whiff test positive

C Wet mount shows clue cell

D White or homogenous grey


vaginal discharge

39.37% People got this right

Explanation:

Correct Answer (A)

pH <4.5

AMSEL CRITERIA is used for the diagnosis of bacterial vaginosis. It is change in


flora with reduction in lactobacilli and increase in anaerobic organisms like
Gardnerella vaginalis. The pH is increased >4.5.

https://emedicoz.com/testresult/683510s14693192 159/176
6/9/24, 12:45 PM Neet PG Preparation, Neet PG Coaching, FMGE, USMLE

(ref: Dutta gynecology, vaginal infections)

Question: 180
One of the following is not included in specific criteria for diagnosis of
PID as per CDC guideline?

A C.Trachomatis or
N.Gonorrhoea: NAAT positive

B Endometrial biopsy
suggestive of endometritis

C TVS showing Tubo-ovarian


abscess

D Laparoscopy showing
hyperaemia and inflammation of
fallopian tube

15.9% People got this right

Explanation:

Correct Answer (A) C.Trachomatis or N.Gonorrhoea: NAAT positive C.Trachomatis


or N.Gonorrhoea: NAAT positive is considered as supportive criteria (not
necessary for the diagnosis). The other three are specific criteria for diagnosis of
PID as per CDC guidelines. (ref: William’s gynecology, genital infections, PID)

Question: 181
Hypertrophic osteoarthropathy and clubbing are paraneoplastic
manifestations of which of the following tumour?

A Ovarian carcinoma

https://emedicoz.com/testresult/683510s14693192 160/176
6/9/24, 12:45 PM Neet PG Preparation, Neet PG Coaching, FMGE, USMLE

B Renal cell carcinoma

C Gastric carcinoma

D Thymoma

13.41% People got this right

Explanation:

Correct Answer (D)

Thymoma. Thymomas have few paraneoplastic syndromes like hypertrophic


osteoarthropathy and clubbing also pure red cell aplasia and myasthenia.

Question: 182
Genetic examination of a non-small cell carcinoma of lung shows
expression of PDL1 and accordingly the patient was started on PDL1
inhibitors. What is the mechanism of action of PDL1 in tumorogenesis?

A Activation of regulatory T
cells

B Activation of downstream
pathways of EGFr

C Activation of ALK protein

D Inhibition of T Cells

14.04% People got this right

Explanation:

Correct Answer (D)

https://emedicoz.com/testresult/683510s14693192 161/176
6/9/24, 12:45 PM Neet PG Preparation, Neet PG Coaching, FMGE, USMLE

Inhibition of T Cells. PD1 and PDL1 expression in tumour cells leads to inhibition
of T cells thus the tumour cells can escape from the host immunity. The PDL1
inhibitors exactly antagonise the same thing. Apart from this tumour cells can
also activate CTLA4 on the T cells which leads to no activation of T cells thus
again allowing the tumour cell to escape from the host immunity.

Question: 183
Match the following Morphological changes with corresponding causes

A A-VI, B-IV, C-I, D-V

B A-IV, B-III, C-II, D-VII

C A-III, B-IV, C-VII, D-II

D A-IV, B-VII, C-I, D-V

42.91% People got this right

Explanation:

Correct Answer (A)

Question: 184
https://emedicoz.com/testresult/683510s14693192 162/176
6/9/24, 12:45 PM Neet PG Preparation, Neet PG Coaching, FMGE, USMLE

A patient presents with pain in the leg. Doppler confirms a thrombus in


the popliteal vein. The patients PT is normal and APTT is increased. Which
of the following is the likely diagnosis?

A Protein C deficiency

B Factor V Leiden mutation

C Factor XII deficiency

D Vitamin K deficiency

31.28% People got this right

Explanation:

Correct Answer (C)

Factor XII deficiency. Elevated APTT with thrombosis is suggestive of Factor XII
deficiency. Factor XII is an activator of plasminogen thus deficiency of the same
leads to thrombotic manifestation.

Question: 185
A 42-year patient diagnosed of Crohn disease for last 3 years. CBC shows
Hb 9 and MCV of 120. What is most likely etiology for such
manifestation?

A Iron loss

B B12 deficiency

C Folic acid deficiency

https://emedicoz.com/testresult/683510s14693192 163/176
6/9/24, 12:45 PM Neet PG Preparation, Neet PG Coaching, FMGE, USMLE

D Both A and B

36.14% People got this right

Explanation:

Correct Answer (B)

B12 deficiency. The patient is having macrocytic anemia and Crohn disease
frequently involves ileum where there is absorption of B12.

Question: 186
Which of the following is increased in Iron deficiency anemia?

A MCHC

B Retic count

C Ferritin

D RDW

44.4% People got this right

Explanation:

Correct Answer (D)

RDW. In IDA RDW is increased due to anisocytosis.

Question: 187
Celiac disease is an example for which type of hypersensitivity?

https://emedicoz.com/testresult/683510s14693192 164/176
6/9/24, 12:45 PM Neet PG Preparation, Neet PG Coaching, FMGE, USMLE

A Type I

B Type II

C Type III

D Type IV

20.07% People got this right

Explanation:

Correct Answer (D)

Type IV.

Question: 188
A 4-year boy has respiratory tract infection, following which he develops
abdominal cramps, joint pain and petechiae on the back. Which of the
following is pathogenesis of the disease?

A Antibodies to
Glycoproteins

B Immune complex mediated


vasculitis

C Antibody mediated
vasculitis

D Disseminated viral infection

41.22% People got this right

https://emedicoz.com/testresult/683510s14693192 165/176
6/9/24, 12:45 PM Neet PG Preparation, Neet PG Coaching, FMGE, USMLE

Explanation:

Correct Answer (B)

The patient is having an Henoch Schoenlein purpura (petechiae with GI and joint
involvement) which is a type III HSR

Question: 189
A patient with barrel shaped chest and jaundice. A liver biopsy is as follows
which of the following disease he is suffering from?

A Hemochromatosis

B Primary biliary cirrhosis

C Alpha 1 antitrypsin
deficiency

D Cystic fibrosis

2.59% People got this right

Explanation:

Correct Answer (B)

https://emedicoz.com/testresult/683510s14693192 166/176
6/9/24, 12:45 PM Neet PG Preparation, Neet PG Coaching, FMGE, USMLE

Alpha 1 antitrypsin deficiency. Liver biopsy shows PAS positive globules


suggestive of Alpha 1 antitrypsin

Question: 190
A gastric biopsy shows small round cells, immunohistochemistry shows
positive for CD20, CD22, CD79a and negative for CD 5, CD 10 and CD23.
Which of the following is the treatment?

A CHOP regimen

B ABVD regimen

C Antibiotics

D Cladribine

8.6% People got this right

Explanation:

Correct Answer (C)

Antibiotics. The markers are suggestive of a post germinal zone lymphoma in


stomach is MALToma which are treated with antibiotics.

Question: 191
Not true about collection of blood is?

https://emedicoz.com/testresult/683510s14693192 167/176
6/9/24, 12:45 PM Neet PG Preparation, Neet PG Coaching, FMGE, USMLE

A Citrate should never be


drawn first

B Green vacutainer for ABG

C Gray tube at last

D Citrated sample for ESR

16.95% People got this right

Explanation:

Correct Answer (A)

Citrate should never be drawn first. Citrate samples are to be drawn at very first
immediately after culture ( if required).

Question: 192
Histology of an ovarian tumors is as follows. What is likely diagnosis?

A Embryonal carcinoma

B Dysgerminoma

https://emedicoz.com/testresult/683510s14693192 168/176
6/9/24, 12:45 PM Neet PG Preparation, Neet PG Coaching, FMGE, USMLE

C Endodermal sinus tumor

D Teratoma

36.9% People got this right

Explanation:

Correct Answer (C)

The microscopy shows Schiller Duval body seen in Endodermal sinus tumor.

Question: 193
Which of the following mutation leads to dilated cardiomyopathy?

A Plakoglobin

B MYH

C MYBPC

D Titin

42.69% People got this right

Explanation:

Correct Answer (D)

Titin.

Option A= Arrhythmogenic dilated cardiomyopathy, option B and C


Hypertrophic cardiomyopathy.

https://emedicoz.com/testresult/683510s14693192 169/176
6/9/24, 12:45 PM Neet PG Preparation, Neet PG Coaching, FMGE, USMLE

Question: 194
Histology of a CNS tumor is as follows which of the following can be
positive in these tumors?

A Cytokeratin

B Progesterone receptor

C GFAP

D S-100

17.14% People got this right

Explanation:

Correct Answer (B)

Progesterone receptor. Histology shows psammoma bodies seen in


meningiomas. Meningiomas are PR positive thus increase in size during
pregnancy

Question: 195
A 53-year female presents with back ache. X ray shows lytic lesion in
thoracic and lumbar vertebra. Serum calcium is increased, total proteins

https://emedicoz.com/testresult/683510s14693192 170/176
6/9/24, 12:45 PM Neet PG Preparation, Neet PG Coaching, FMGE, USMLE

are increased, and albumin is normal. Which of the following is likely


diagnosis?

A Metastatic breast
carcinoma

B Langerhans cell
histiocytosis

C Osteoporosis

D Multiple myeloma

41% People got this right

Explanation:

Correct Answer (D)

Multiple myeloma. Total proteins are increased but albumin is normal indicated
that the patient is having increased globulins along with lytic lesion and hyper
calcemia the diagnosis is multiple myeloma.

Question: 196
Which of the following Implant terminology doesn't match with its
Mechanism of Action?

https://emedicoz.com/testresult/683510s14693192 171/176
6/9/24, 12:45 PM Neet PG Preparation, Neet PG Coaching, FMGE, USMLE

A Plates: Load Bearing


Devices

B Nails: Load sharing Devices

C Ilizarov's Fixator:
Distraction Histiogenesis

D Steinmann's Pin: Three


Point Fixation

30.62% People got this right

Explanation:

Correct Answer (D)

Steinmann's Pin : Three Point Fixation

Steinmann's Pain is the most common orthopaedic implant for application of


skeletal traction overall. It is Kuntschner's Nail (K Nail ) which is used for
Intramedullary fixation of long bone ( femur/ tibia ) shaft fracture and that works
on three point fixation.

Question: 197
A 7year old boy is brought by parents to Ortho OPD with chief complains
of dull aching pain and inability to squat. His problems started with
intense pain in the right hip two weeks ago for which he was treated
conservatively and symptomatically. His blood investigations show raised
ESR with other counts almost normal. X rays were done which showed
osteopenia and mild erosions around the hip joint mainly towards the
acetabular side. Which of the following is the correct diagnosis?

A Perthes' disease

https://emedicoz.com/testresult/683510s14693192 172/176
6/9/24, 12:45 PM Neet PG Preparation, Neet PG Coaching, FMGE, USMLE

B Septic Arthritis

C Tuberculosis hip

D Slipped Capital Femoral


Epiphysis

19.12% People got this right

Explanation:

Correct Answer (C)

Tuberculosis hip

His problems started with intense pain in the right hip two weeks ago for which
he was treated conservatively and symptomatically:

Stage I TB hip

X rays were done which showed osteopenia and mild erosions around the hip
joint mainly towards the acetabular side: Stage II/III TB Hip

Had it been Perthes', X rays would have shown destruction of head of femur
rather than acetabular involvement.

Question: 198
An 8 year old child sustains fall from height while flying kite on the roof.
He develops gross swelling over right ankle immediately. He is taken to a
doctor whereby X-rays were done which don't show any bony fracture.
The child is kept on conservative management. Two years later, the child
develops Calcaneovalgus deformity of the ankle. What did we miss Two
years ago?

A Salter Harris V Physeal


Trauma

https://emedicoz.com/testresult/683510s14693192 173/176
6/9/24, 12:45 PM Neet PG Preparation, Neet PG Coaching, FMGE, USMLE

B Avascular Necrosis of Talus

C Secondary Osteoarthritis
Ankle

D Fracture Calcaneum

29.47% People got this right

Explanation:

Correct Answer (A)

Salter Harris V Physeal Trauma

This is a classical case of Salter Harris V Physeal Trauma where there is partial or
complete crushing injury to the physis. Since there is cartilaginous injury, its not
usually evident initially on X rays.

Question: 199
Identify the test being performed in this image

A Finkelstein's Test

B Phalen's test

https://emedicoz.com/testresult/683510s14693192 174/176
6/9/24, 12:45 PM Neet PG Preparation, Neet PG Coaching, FMGE, USMLE

C Durkan's Test

D Adson's Test

47.59% People got this right

Explanation:

Correct Answer (A)

Finkelstein's test is a test used to diagnose de Quervain's tenosynovitis in people


who have wrist pain. Classical descriptions of the Finkelstein's test are when the
examiner grasps the thumb and ulnar deviates the hand sharply.

Question: 200
A 26 year old male has been complaining of back pain for last one year.
His pain radiates from the back to right buttock and thigh down to legs.
On examination he has mild sensory loss over dorsum of foot and
weakness of extension of great toe on the right side. Which of the
following is the most probable diagnosis ?

A PIVD L4-L5 Postero lateral /


Paracentral

B PIVD L5-S1 Postero central

C PIVD L4-L5 Far Lateral /


Foraminal

D PIVD L5-S1 Postero lateral /


Paracentral

17.83% People got this right

Explanation:

https://emedicoz.com/testresult/683510s14693192 175/176
6/9/24, 12:45 PM Neet PG Preparation, Neet PG Coaching, FMGE, USMLE

Correct Answer (A)

Dermatome and myotome explained in the question means L5 So it will be


either Posterolateral / Paracentral L4-L5 Disc Prolapse or Far Lateral/ Foraminal
L5 -S1 Disc Prolapse or both

https://emedicoz.com/testresult/683510s14693192 176/176

You might also like